+7 495 120-13-73 | 8 800 500-97-74

(для регионов бесплатно)

Содержание

Урок 46. Лабораторная работа № 12. Измерение индуктивности катушки.

Тема: Измерение индуктивности катушки

Цель: вычисление индуктивного сопротивления катушки и ее индуктивности по результатом измерений напряжений на катушке и силы тока в цепи.

Оборудование: источник переменного напряжения; катушка школьного разборного трансформатора; вольтметр и миллиамперметр переменного тока; соединительные провода.

Теория.

   Всякое изменение тока в катушке вызывает появление в ней ЭДС самоиндукции, препятствующей изменению тока. Величина ЭДС самоиндукции прямо пропорциональна величине индуктивности катушки и скорости изменения тока в ней. Но так как переменный ток непрерывно изменяется, то непрерывно возникающая в катушке ЭДС самоиндукции создает сопротивление переменному току. Она препятствует его возрастанию и, наоборот, поддерживает его при убывании. Таким образом, в катушке индуктивности, включенной в цепь переменного тока, создается сопротивление прохождению тока.

Но так как такое сопротивление вызывается в конечном счете индуктивностью катушки, то и называется оно индуктивным сопротивлением.

   Индуктивное сопротивление обозначается через ХL и измеряется, как и активное сопротивление, в омах. Индуктивное сопротивление цепи тем больше, чем больше частота тока, питающего цепь, и чем больше индуктивность цепи. Следовательно, индуктивное сопротивление цепи прямо пропорционально частоте тока и индуктивности цепи; определяется оно по формуле:

 ХLL

 , где ω — круговая частота, определяемая произведением 2πν, L — индуктивность цепи в генри (Гн).

   Т.е.

   Тогда индуктивность катушки можно выразить:

   Закон Ома для цепи переменного тока, содержащей индуктивное сопротивление, звучит так: величина тока прямо пропорциональна напряжению и обратно пропорциональна  индуктивному сопротивлению цепи, т. е

   , где I и — действующие значения тока и напряжения, а ХL — индуктивное сопротивление цепи. 

Выполнение работы:

1. Подготовить таблицу для результатов измерений и вычислений:

Напряжение
U, В

Сила тока
I, мА

Индуктивное сопротивление
XL, Ом

Частота
ν, Гц

Индуктивность

L, мГн

 

 

 

 

 

 

 

 

 

 

 

 

 

 

 

 

 

 

 

 

 

 

 

 

 

 

 

 

 

 

2. Собрать электрическую схему согласно рисунка 1 и перечертить её в тетрадь:

3. Спомощью регулятора напряжения подать на схему напряжение 1,5 В и установить частоту переменного тока 80 Гц. Записать показания миллиамперметра.

4. Увеличивая частоту в 2,3,4 и 5 раз каждый раз записывать показания миллиамперметра в таблицу.

5. Вынуть сердечник из катушки и, не изменяя напряжения и частоты переменного тока, записать показания миллиамперметра в таблицу.

Напряжение
U, В

Сила тока
I, мА

Индуктивное сопротивление
XL, Ом

Частота
ν, Гц

Индуктивность
L, мГн

 1,5

 0,345

 

 80

 

 1,5

 0,178

 

 160

 

 1,5

 0,121

 

 240

 

 1,5

 0,090

 

 320

 

 1,5

 0,072

 

 400

 

 1,5

 0,284

 

 400

 

6. В каждом опыте рассчитать индуктивное сопротивление катушки по формуле:

7. Вычислить в каждом опыте индуктивность катушки L, используя формулу:

8. Сравнивая индуктивности катушек, сделайте вывод, от чего и как зависит индуктивность.

9. Ответьте письменно на контрольные вопросы.

Контрольные вопросы.

1. Чем вызвано индуктивное сопротивление у катушки при подключении её в цепь переменного тока?

2. От чего зависит индуктивное сопротивление?

3. Почему уменьшается индуктивное сопротивление при удалении из катушки железного сердечника?

4. Почему на постоянном токе индуктивное сопротивление катушки равно нулю?

5. Чему равно индуктивное сопротивление в цепи переменного тока?

6. Как связаны между собой действующие значения силы тока и напряжения на катушке индуктивности?

    

Калькулятор импеданса катушки индуктивности • Электротехнические и радиотехнические калькуляторы • Онлайн-конвертеры единиц измерения

Отметим, что величина импеданса идеальной катушки индуктивности равна ее реактивному сопротивлению. Однако это не идентичные величины, так как между током и напряжением в индуктивной цепи существует фазовый сдвиг. Для расчетов используются указанная ниже формула:

Здесь

XL — реактивное сопротивление катушки в омах (Ом),

ZL — импеданс катушки в омах (Ом),

ω = 2πf — угловая частота в рад/с,

f — частота в герцах (Гц),

L — индуктивность в генри (Гн),

j — мнимая единица.

Для расчета выберите единицы измерения и введите индуктивность и частоту. Импеданс катушки индуктивности будет показан в омах.

Катушка индуктивности представляет собой пассивный электрический элемент с двумя выводами, изготовленный, как правило, из изолированного провода в форме спирали, намотанного на магнитный сердечник или на оправку (без сердечника). Магнитный сердечник обычно изготовляется из ферромагнитного металла, например, железа или ферромагнитной керамики (феррита) и используется для усиления магнитного поля и, таким образом, для увеличения индуктивности катушки. Как и конденсаторы, катушки индуктивности используются для накопления и сохранения энергии. Однако, в отличие от конденсаторов, энергия в катушке хранится в форме окружающего ее магнитного поля. Катушки индуктивности применяются, в частности, в фильтрах для сглаживания постоянного тока или для предотвращения передачи высокочастотных помех по кабелям. Катушки индуктивности широко используются в колебательных контурах радиопередатчиков и радиоприемников, а также для изготовления трансформаторов.

Высокодобротная катушка индуктивности без сердечника, установленная в радиопередатчике

В отличие от конденсаторов, которые препятствуют изменению напряжения, приложенного к их обкладкам, катушки индуктивности препятствуют изменению текущего в них тока. В отличие от конденсаторов, которые не пропускают постоянный ток, катушки индуктивности пропускают его легко. Они препятствуют только прохождению переменного тока или иного изменяющегося тока и их способность препятствовать прохождению тока и называется индуктивностью. Индуктивность обозначается символом L в честь российского физика Эмиля Ленца и измеряется в генри — единицах, названных в честь американского ученого Джозефа Генри.

В отличие от резисторов, которые препятствуют прохождению электрического тока вследствие падения напряжения на них, пропорционального протекающему току, катушки индуктивности препятствуют изменению протекающего через них тока. На них создается падение напряжения, прямо пропорциональное скорости изменения тока. Полярность индуцированного напряжения всегда такова, что это напряжение поддерживает изменяющийся ток в его текущем состоянии. Например, если ток растет, то напряжение препятствует этому росту и старается уменьшить ток. В то же время, если ток через катушку уменьшается, напряжение мешает этому уменьшению и поддерживает более высокий ток. Чем выше скорость изменения тока, тем больше амплитуда этого обратного напряжения. В связи с указанным свойством, это напряжение часто называют индуктивными выбросами или обратным током. Для того, чтобы как-то отличить это свойство от сопротивления, используют понятие реактивного сопротивления. Если к катушке индуктивности приложено синусоидальное напряжение, то при бóльших частотах катушка оказывает току большее сопротивление, следовательно, ее реактивное сопротивление также увеличивается, как показано на графике.

График зависимости реактивного сопротивления катушки XL индуктивности и текущего через нее тока I от частоты f для нескольких величин индуктивности показывает прямую пропорциональную зависимость от частоты реактивного сопротивления и обратную зависимость от частоты протекающего через катушку тока

Полное сопротивление Z, как и реактивное сопротивление, измеряется в омах (Ом) и состоит из двух частей — действительной и мнимой. Первая из них представляет собой активное сопротивление R, которое затрудняет протекание тока в материале с плохой проводимостью и зависит от формы этого материала. Вторая часть — это рассмотренное выше реактивное сопротивление X. Оно также затрудняет протекание тока, но не вследствие свойств и формы проводящего материала, а из-за того, что протеканию тока мешает электрическое и магнитное поля.

Если реальная катушка индуктивности подключена к источнику постоянного напряжения, через нее протекает постоянный ток, ограниченный только сопротивлением провода, из которого намотана катушка. Когда катушка подключается к источнику постоянного напряжения, ток через нее медленно повышается от нуля до максимального значения, которое определяется внутренним сопротивлением источника и внутренним сопротивлением витков катушки. ЭДС самоиндукции, возникающая в катушке, препятствует резкому увеличению тока в ней. Эта ЭДС противодействует приложенному напряжению до тех пор, пока ток не достигнет максимального значения.

Если источник постоянного напряжения отключить от катушки, протекающий через нее ток постепенно падает до нуля. В этом случае опять возникает ЭДС самоиндукции, которая снова препятствует, на этот раз, уменьшению тока и которая старается поддержать ток в неизменном состоянии. В конце концов, ток постепенно уменьшается до нуля.

В чисто индуктивной цепи ток отстает от напряжения на π/2 радиан или 90°. 1 — в момент отрицательного максимума тока скорость его изменения нулевая и напряжение равно нулю; 2 — ток нулевой, скорость его изменения максимальная и напряжение равно положительному максимуму; 3 — ток положительный и максимальный, скорость его изменения нулевая и напряжение равно нулю; 4 — ток нулевой, скорость его изменения максимальная и напряжение равно отрицательному максимуму

Если к катушке индуктивности приложено переменное синусоидальное напряжение, ток отстает от напряжения на некоторый фазовый угол, как показано на графике. В идеальной катушке индуктивности этот угол будет равен точно 90° или четверти цикла. В точке (ωt = π/2), временнóй оси, где ток нулевой, напряжение на катушке достигает своего положительного максимума. Затем ток постепенно увеличивается и вокруг катушки создается магнитное поле, которое, в свою очередь, создает ЭДС самоиндукции, направленную противоположно току. Эта ЭДС является реакцией катушки на изменение протекающего через нее тока, и она максимальна, когда ток нулевой, так как в этой точке скорость изменения тока максимальная. Когда же ток достигает своего максимального значения (положительного или отрицательного), скорость изменения синусоидального тока становится нулевой и в этих точках максимумов ток ЭДС самоиндукции (определяемая этой скоростью) также равна нулю. Это приводит к тому, что синусоида напряжения не совпадает по фазе с током на угол 90° или π/2 радиан. То есть, напряжение опережает ток или ток отстает от напряжения.

Аналогичное явление можно наблюдать и в природе. Сравните: Солнце светит сильнее всего в астрономический полдень (солнечный свет — напряжение), однако самая жаркая часть дня обычно бывает через несколько часов после полудня (температура — ток). Или другой пример. День зимнего солнцестояния в северном полушарии (самый короткий день) — в конце декабря, однако самые холодные месяцы еще впереди. В зависимости от того, где вы живете, это будет январь или февраль. Вспомните поговорку «Солнце — на лето, зима — на мороз». Это как раз о поведении индуктивности, только в природной аналогии. Такой сезонный «сдвиг фаз» или отставание вызван поглощением энергии Солнца огромными массами воды в океанах. Они отдадут эту запасенную энергию, но позже — точно так же, как это делают катушки индуктивности.

День зимнего солнцестояния в северном полушарии — в конце декабря, однако самые холодные месяцы еще впереди. Именно так ведет себя ток в катушке индуктивности

Рассчитанный этим калькулятором импеданс представляет собой меру сопротивления катушки индуктивности протекающему через нее току на определенной частоте. Индуктивное реактивное сопротивление изменяется при изменении частоты приложенного переменного напряжения. Приведенные выше формула и график показывают, что реактивное сопротивление катушки индуктивности XL велико при высоких частотах и мало при низких частотах (конденсаторы ведут себя с точностью до наоборот). При высоких частотах индуктивное реактивное сопротивление становится очень большим и очень сильно противодействует протекающему току. С другой стороны, при очень низких частотах или при постоянном напряжении катушка индуктивности проводит очень хорошо — отсюда правило, которое мы выучили в школе: катушки индуктивности не пропускают переменный ток и пропускают постоянный. Если частота очень низкая, катушки индуктивности пропускают сигнал очень хорошо. Именно поэтому их устанавливают в фильтрах громкоговорителей (кроссоверах), чтобы высокие частоты не попадали на низкочастотные динамические головки.

Импеданс измеряется в омах, так же, как и сопротивление. Импеданс мешает прохождению электрического тока так же, как и сопротивление, и показывает как сильно катушка противодействует прохождению тока через нее. Но тогда возникает вопрос: в чем же разница между импедансом и сопротивлением? А разница заключается в зависимости импеданса от частоты приложенного сигнала. Сопротивление от частоты не зависит, а импеданс катушек индуктивности от частоты зависит. С увеличением частоты импеданс катушек уменьшается.

Этот калькулятор предназначен для расчета импеданса идеальных катушек индуктивности. Реальные катушки всегда имеют сопротивление, которое на эквивалентной схеме изображают включенным последовательно с индуктивностью. Для расчета импеданса реальных катушек индуктивности пользуйтесь калькулятором импеданса RL-цепей.

Катушки индуктивности в высокочастотном модуле телевизионного приемника

Автор статьи: Анатолий Золотков

Сопротивление цепи переменного тока индуктивное

Индуктивный датчик представляет собой катушку с железным сердечником, включенную в цепь переменного тока. Индуктивность такой катушки определяется ее магнитным сопротивлением. Последнее же является функцией трех величин длины сердечника, площади его поперечного сечения и магнитной проницаемости его материала. Если датчик имеет якорь, замыкающий магнитную цепь катушки и отделенный от сердечника небольшим зазором, то магнитное сопротивление будет в основном зависеть от характеристик зазора его величины, поперечного сечения и магнитной проницаемости.
[c.37]
Найдите резонансную частоту последовательной цепи переменного тока конденсатора емкостью 10 мкФ и катушки индуктивностью 1 Гн с активным сопротивлением 10 Ом.  [c.296]
Фиг. 7. Цепь переменного тока, содер жащая последовательно включенные активное, индуктивное и емкостное сопротивления.
Принцип работы индуктивных измерительных приборов заключается в том, что с изменением размера контролируемого изделия изменяется воздушный зазор в замкнутом дросселе и сопротивление в цепи переменного тока. Электросхема прибора представляет собой мостовую схему. Измеряемая величина находится в определенной зависимости от тока, протекающего в цепи и выпрямленного для измерения, сортировки или регулирования необходимые управляющие процессы осуществляются с помощью специального реле. Ввиду того, что магнитная цепь индуктивных преобразователей обладает очень малыми воздушными зазорами, незначительное изменение измеряемой величины соответствует сравнительно большому изменению магнитного сопротивления. Существенным преимуществом индуктивных приборов для контроля размеров является отсутствие в преобразователе чувствительных опор, шарниров, контактов, которые вызывают чувствительность прибора к сотрясениям, ограничивают его надежность и срок службы при эксплуатации.  [c.215]

Магнитный усилитель представляет собой электромагнитное устройство, в котором с помощью сигнала постоянного тока осуществляется управление значительно большей мощностью переменного тока. На рис. 31, а представлен магнитный усилитель на двух сердечниках 2 и 3 с общей управляющей обмоткой 5, намотанной на оба сердечника. Обмотка 5 присоединена к цепи постоянного тока, а обмотки 4п 1 — к цепи переменного тока. Небольшие изменения силы постоянного тока в обмотке 5 меняют индуктивное сопротивление и силу тока в обмотках 4и I. Магнитные усилители виброустойчивы, дешевы, имеют большой коэф-  [c.163]


Предельный коэффициент эффективности акустического излучения. В цепях переменного тока с последовательным соединением мощность, расходуемая источником э.д. с., идет на нагревание активного сопротивления. Индуктивная нагрузка накапливает энергию в форме энергии магнитного поля и периодически обменивается ею с источником напряжения. Аналогичный процесс осуществляется и в поле при излучении акустических волн мощность источника энергии излучателя поглощается в виде потока энергии аку-  [c.200]

Катушка индуктивности Ь обладает индуктивным сопротивлением, т. е. сопротивлением, которое вносит в цепь переменного тока катушка индуктивности вследствие явления самоиндукции.  [c.97]

Таким образом, мощность, связанная с реактивной частью импеданса, аналогична мощности, потребляемой индуктивностью в цепи переменного тока, а сама реактивная часть 1т 2 — индуктивному сопротивлению катушки. Активная же часть Не 2 = р с ЗоЯ определяет мощность, необратимо теряемую источником на излучение в среду, и она эквивалентна активному сопротивлению электрической цепи. Поэтому эквивалентная схема акустического импеданса пульсирующей сферы может быть представлена параллельно соединенными катушкой и омическим сопротивлением.  [c.208]

В цепь переменного тока при этом включены электромагнитные вентили ОЭ и ТЭ электровоздухораспределителей ЭВ, однако они не сработают из-за большого индуктивного сопротивления их катушек.  [c.63]

Магнитные усилители и дроссели насыщения могут рассматриваться как регулируемые индуктивные сопротивления, включаемые в цепь переменного тока. Изменения величины индуктивного сопротивления магнитного  [c.67]

Основное свойство дросселя насыщения состоит в том, что величина реактивного (индуктивного) сопротивления обмоток переменного тока зависит от величины постоянного тока в обмотке управления. Это свойство объясняется способностью стали насыщаться. При насыщении сердечника уменьшается его магнитная проницаемость, от которой зависит индуктивность обмоток. Если в обмотке управления нет тока, сопротивление рабочих обмоток будет большим и ток в цепи рабочие  [c.358]

Если цепь переменного тока содержит, кроме активного сопротивления, также и индуктивное сопротивление, то напряжение и ток не совпадают по фазе. В этом случае, в зависимости от соотношения между индуктивным и активным сопротивлением, ток будет отставать по фазе от напряжения на тот или другой угол.  [c.35]

Схема включения простейшего индуктивного датчика приведена на рис. 82, а. Дроссель индуктивного датчика ДИ включен в цепь переменного тока последовательно с исполнительным токовым реле Р. Когда магнитная цепь датчика разомкнута, т. е. когда напротив П-об-разного сердечника нет магнитного шунта, индуктивное сопротивление датчика мало и исполнительное реле Р включено. Когда магнитный шунт, укрепленный на кабине, подходит к датчику, закрепленному в шахте, магнитная цепь датчика замыкается, его индуктивное сопротивление резко возрастает и исполнительное реле отключается вследствие уменьшения тока в цепи датчика. Работа индуктивного датчика не меняется, если дроссель датчика укреплен на кабине, а магнитный шунт — в шахте.  [c.121]

Для регулирования тока возбуждения генератора применен однофазный магнитный усилитель. Силовые обмотки магнитного усилителя ОС/ и 0С2 включены в цепь переменного тока совместно с селеновым выпрямителем Вп2 и обмоткой возбуждения генератора Г таким образом, что по обмотке Г течет постоянный ток, величина которого зависит от реактивного (индуктивного) сопротивления обмоток 0С1 и 0С2. Чем меньше сопротивление обмоток, тем больше величина тока в обмотке возбуждения генератора. Когда все обмотки управления магнитного усилителя выключены, сопротивление силовых обмоток так велико, что в обмотке возбуждения тормозного генератора практически нет тока.[c.184]


Основное свойство дросселя насыщения состоит в том, что величина реактивного (индуктивного) сопротивления обмоток переменного тока зависит от величины постоянного тока в обмотке управления, Это свойство объясняется способностью стали насыщаться. При насыщении сердечника уменьшается его магнитная проницаемость, от которой зависит индуктивность обмоток. Если в обмотке управления нет тока, сопротивление рабочих обмоток будет большим и ток в цепи рабочие обмотки — потребитель будет иметь наименьшее значение. С появлением тока в цепи обмотки управления реактивное сопротивление рабочих обмоток уменьшится, следовательно, возрастет ток в цепи потребителя.  [c.121]

Индуктивное сопротивление в цепи переменного тока  [c.36]

Активное, индуктивное и емкостное сопротивления в цепи переменного тока  [c.36]

В цепях переменного тока различают активное, индуктивное и емкостное сопротивления.[c.12]

СВАРОЧНЫЙ ДРОССЕЛЬ (для дуговой сварки) — регулируемое индуктивное сопротивление, включаемое последовательно с дугой в сварочную цепь переменного тока.  [c.146]

Магнитные усилители. Магнитным усилителем называется электромагнитный аппарат, в котором для плавного регулирования переменного тока изменяют индуктивное сопротивление катушки с сердечником путем подмагничивания ее постоянным током. До того как перейти магнитному усилителю, остановимся на дросселе насыщения (рис. 65, а), состоящем из ферромагнитного сердечника с катушкой индуктивности 1 (рабочей) и подмагничивающей катушкой 2 (управления). Если включить дроссель в -цепь переменного тока и изменять ток управления /у, то будет изменяться индуктивность в рабочей обмотке 1 и создаваемое ею сопротивление, а следовательно, будет изменяться ток в цепи нагрузки.  [c.122]

В сварочную цепь переменного тока (рис. 96, 6) включено омическое и индуктивное сопротивление (балластный реостат и дроссель)  [c. 153]

В сварочную цепь переменного тока (рис. 98, б) включено омическое и индуктивное сопротивления (балластный реостат и  [c.163]

На фиг. 168, а изображен датчик с малым воздушным зазором 8, длина которого изменяется под действием измеряемой механической величины Р. Вследствие изменения зазора, изменяется магнитное сопротивление магнитной цепи, а следовательно, и индуктивность катушки, надетой на сердечник и включенной в цепь переменного тока. Изменение индуктивного сопротивления катушки ведет к изменению ее полного сопротивления Z.  [c.210]

Общепринятая мера стабилизации сварочной дуги переменного тока — включение в сварочные цепи переменного тока дросселей, что позволяет поддерживать стабильность дуги и регулировать сварочный ток изменением индуктивного сопротивления.  [c.31]

Распространение волн по разветвленной системе можно, как мы видели, удобно описать, если представить себе произвольную волну разложенной на компоненты, пропорциональные е , и использовать комплексную проводимость У, зависящую от ю, для определения отклика любой части системы на такие компоненты. Общая формула, которая, если пренебречь ослаблением волны, имеет вид (61), связывает эффективную проводимость у предыдущего разветвления с проводимостями у последующего разветвления. Многократное применение этой формулы в обратном порядке, начиная от наиболее отдаленных разветвлений и кончая самым первым, позволяет охарактеризовать свойства всей системы подобным образом цепи переменного тока изучаются с помощью суммирования (в соответствии с законами Кирхгофа) зависящих от частоты комплексных проводимостей (или сопротивлений) сосредоточенных элементов сети. Эта аналогия вызывает вопрос, могут ли для одномерных волн в жидкости существовать какие-либо сосредоточенные элементы с чисто мнимой проводимостью, подобные таким обычным элементам электрической цепи, как емкости и индуктивности. В этом разделе мы найдем их близкие аналоги, укажем, как можно проанализировать системы с такими элементами, и исследуем условия резонанса, в некоторых случаях аналогичные условиям колебательного контура .[c.144]

Индуктивность в цепи переменного тока. В любом проводнике, по которому протекает переменный ток, возникает ЭДС самоиндукции. Поэтому ни одна электрическая цепь не обладает только актикным сопротивлением.  [c.242]

В цепях переменного тока рассеяние мощности в катушках индуктивности иногда оценивают тангенсом угла магнитных потерь. Тороидальную катушку индуктивности с сердечником из магнитного материала, собственной емкостью и сопротивлением обмотки 1чОторой можно пренебречь, представим в виде схемы, состоящей из последовательно соединенных индуктивности L и сопротивления 1квивалентн0г0 всем видам потерь мощности в магнетике (рис. 9-10) для этого случая из векторной диаграммы получим  [c.273]

Электрическое сопротивление активное, реактивное и полное (комплексное). В цепи переменного тока различают активное и реактивное сопротивления. Первым обладает участок цепи, в котором отсутствует индуктивность или емкость. Реактивное сопротивление может быть индуктивным, равным о)(где Ь — индуктивность, а со — круго-  [c.247]

Новый класс частотозависимых мостовых цепей переменного тока, уравновешиваемых изменением одной лишь частоты, позволил разработать универсальные высокочастотные преобразователи сопротивления, емкости и индуктивности в частоту переменного тока и компенсационные частотомеры. Аналоговые мосты и компенсаторы переменного тока с непрерывным автоматическим уравновешиванием двумя параметрами, позволяющие одновременно измерять, контролировать и регулировать обе составляющие комплексных величин, были разработаны в период с 1956 по 1960 г.  [c.263]


Резонанс. Явления резонанса возникают в цепях переменного тока при равенстве индуктивного и ёмкостного сопротивлений или при равенстве индуктивной и ёмкостной проводимости. В этих случаях контур по отношению внешней цепи является безиндуктивным, как бы состоящим из одного активного сопротивления.[c.521]

ИНДУКТИВНОЕ СОПРОТИВЛЕНИЕ в цепи переменного тока — реактивная часть сопротивления двухполюсника (см. Импеданх), в к-рои синусоидальный ток отстаёт по фазе от приложенного напряжения подобно тому, как это имеет место для катуш- КН самоиндукции. В идеальном случае, когда катушка самоиндукции может быть охарактеризована единств, параметром — индуктивностью i = onst, И. с. определяется как отношение амплитуд напряжепия и тока и равно Xi — aL (oj —- циклич. частота). При этом ток отстаёт по фазе от напряжения точно на угол я/2, вследствие чего в среднем за период но происходит ни накопления эл.-магп. энергии в катушке, ип её диссипации дважды за период энергия накачивается внутрь катушки (в основном в виде энергии маги, поля) и дважды возвращается обратно источнику (или во внеш. цепь).  [c.141]

Индуктивным преобразователем (датчиком) является электромагнитное устройство, преобразующее контролируемую неэлектрическую величину (перемещение уровня ванны) в электрический параметр (индуктивное сопротивление). Простейщий индуктивный датчик представляет собой магнитную цепь, состоящую из сердечника с катущкой и подвижного якоря, разделенных воздущным зазором 6 (рис. 3.15). Полное сопротивление катущки со стальным сердечником в цепи переменного тока  [c.161]

Применение тиристорной схемы управления позволяег простыми средствами бесступенчато регулировать обороты электродвигателя. Диоды Д7—Д10 в цепи управления тиристоров Д5 и Д6 установлены для предотвращения возникновения импульса обратной полярности на управляющем электроде. Резисторы R1 и R2 включены для выравнивания углов зажигания тиристоров. Сдвиг фазы управляющего напряжения относительно напряжения питания тиристоров осуществляется с помощью фазовращателя, который представляет собой цепь переменного тока, содержащую активное, индуктивное и емкостное сопротивления. При изменении сопротивлений резисторов R3 и R4 фаза управляющего напряжения тиристоров сдвигается в идеальном случае от О до 180°, практически л[c. 332]

Изменение тока в электрической цепи (включение, выключение) вызывает появление в ней ЭДС самоиндукции, препятствующей этому изменению. При увеличении тока она направлена против ЭДС источника напряжения, а при уменьшении тока, она мешает ему исчезнуть. Сопротивление в цепи, возникающее в результате действия ЭДС самоиндукции, называется индуктивным, а сопро-тивл 1ние проводников цепи—активным. Вся мощность, получаемая цепью переменного тока, называется кажущейся и состоит из активной и реактивной — мощностей. Активная мощность расходуется на нагрев. В двигателях переменного тока большая часть активной мощности превращается в механическую. Реактивная мощность обусловлена наличием магнитных и электрических полей в индуктивностях и емкостях цепей. В цепи с индуктивной нагрузкой нельзя избежать наличия реактивной мощ-  [c.31]

V.4.29. Реактивное сопротивление (реоктанс) электрической цепи переменного тока (при последовательном соединении индуктивности L и емкости О  [c. 58]

В любой электрической цепи переменного тока вокруг проводников с током возникает магнитное поле, следовательно электрическая цепь всегда обладает индуктивностью. Если переменное напряжение приложить к катушке индуктивности, ток в цепи будет меньше в сравнении с тем током, который бы протекал при наличии одного активного сопротивления катуш ки. ЭДС самоиндукции катушки противодействует периодическим изменениям переменного тока, т. е. в катушке возникает дополнительное препятствие (кроме активного сопротивления) прохождению по ней переменного тока. Противодействие катушки индуктивности переменному току, измеряемое в омах, условно назвали индуктивным сопротивлением Индуктивное сопротивление пропорционально индуктивности цепи и частоте переменного токя Xц=2n f L. Коэффициент 2л  [c.12]

П. переменного тока. При компенсации на переменном токе необходимо, чтобы непосредственно сравниваемые эдс были равны по величине и имели одинаковые 1) частоту, 2) форму кривой и 3) фазу. Выполнения первых двух условий достигают, питая потенциометр через соответствующий трансформатор от того же генератора, напряжение к-рого нужно измерить. Для выполнения третьего условия необходим регулятор фаз (П. сист. Дрисдаля) или особый трансформатор без железа (комплексный П. системы Гартмана и Брауна). В виду отсутствия эталона переменной эдс для установления силы рабочего тока в П. переменного тока служат электродинамические амперметры, поэтому точность измерения величины напряжения не превосходит точности этого амперметра (0,5%). П. переменного тока применяются при всех точных измерениях в цепях переменного тока при калибровке амперметров и вольтметров, при точном измерении емкостного и индуктивного сопротивления цепи, при определении угла сдвига фаз между токами в отдельных участках цепи. Измерение угла при помощи регулятора фаз м. б. произведено с точностью не более 0,5°, с помощью комплексного П.—до 0,25°, но измерение последним величины эдс имеет погрешность 0,5  [c. 241]

Обозначения h(H) — высота оси вращения i3jj — наружный диаметр сердечников статоров (для асинхронных двигателей) Р — номинальная мощность 7 — номинальное напряжение питания /ц —номинальное значение силы тока — номинальная частота вращения вала — номинальный момент max — максимальная частота вращения вала т — коэффициент полезного действия Ля — сопротивление якорной обмотки Лд — сопротивление дополнительных полюсов (на дополнительных полюсах располагается компенсационная обмотка, которая включается последовательно с обмоткой якоря и предназначена для улучшения процесса коммутации в щеточно-коллекторном узле) — сопротивление обмотки возбуждения — индуктивность обмотки якоря J — момент инерции якоря S — номинальное скольжение М ах> — максимальный и пусковой момент на валу соответственно (для асинхронных двигателей) — пусковой ток os ф — коэффициент мощности (отношение активной мощности цепи переменного тока к полной мощности, чем ближе к единице, тем лучше).  [c.194]

Магнитные усилители. В настоящее время наряду с электронными усилителями в САУ широко применяют магнитные усилители. Основой магнитного усилителя является дроссель насыщения, представляющий собой реактивную катушку с двумя обмотками и сердечником из ферромагнитного материала. Одна из обмоток включается в цепь переменного тока, а другая используется для подмагничивания и обтекается постоянным током. При прохождении по обмотке постоянного тока /= (ток подмагничивания) в сердечнике возникает постоянное магнитное поле Н=, что приводит к уменьшению магнитной проницаемости сердечника для обмотки переменного тока. В результате этого снижается индуктивное сопротивление дросселя = = (о1/др(1/др ц) и переменный ток 1 = /-//соЬдр возрастает. Таким образом, изменением силы тока в обмотке подмагничивания можно управлять силой тока в обмотке переменного тока. Этот принцип лежит в основе работы магнитного усилителя.  [c.890]


Принцип работы индуктивных измерительных приборов заключается в том, что с изменением размера контролируе.мого изделия изменяется воздушный зазор в замкнутом дросселе и вместе с тем сопротивление в цепи переменного тока. Электросхеыа прибора представляет собой мостовую схему. Измеряемая величина находится в определенной зависимости от тока, протекающего в цепи и выпрямленного для целей измерения, целей сортировки или регулирования необходимые управляющие процессы осуществляются с помощью лампового каскада или специального реле. Ввиду того, что raгнитнaя цепь индуктивных датчиков обладает очень малыми воздушными зазорами, весьма незначительное изменение измеряемой величины соответствует сравнительно большому изменению магнитного сопротивления. Следовательно, в индуктивных измерительных приборах можно обойтись без рычажной передачи перемещение измерительного штока передается непосредственно на воздушный зазор в магнитной цепи. В некоторых конструкциях индуктивных приборов применяют односторонний якорь, закрепленный в пружинном шарнире. Существенным преимуществом индуктивных приборов для контроля размеров является отсутствие в датчике чувствительных опор, шарниров, контактов, которые вызывают чувствительность прибора к сотрясениям, ограничивают его надежность и срок службы при эксплуатации.  [c.440]

2. Катушка индуктивности в цепи переменного тока | 3. Реактивное сопр. и импеданс — Индуктивность | Часть2

2. Катушка индуктивности в цепи переменного тока

Катушка индуктивности в цепи переменного тока

Катушка индуктивности в цепи переменного тока ведет себя не так, как резистор. Если резисторы просто противостоят потоку электронов (напряжение на них прямопропорционально току), то катушки индуктивности противостоят изменению проходящего через них тока (напряжение на них прямопропоционально скорости изменения тока). Согласно Закону Ленца, индуцированное напряжение всегда имеет такую полярность, которая пытается сохранить текущее значение силы тока. То есть, если величина тока возрастает, то индуцированное напряжение будет «тормозить» поток электронов; если величина тока уменьшается, то полярность напряжения развернется и будет «помогать» электронному потоку оставаться на прежнем уровне. Такое противостояние изменению величины тока называется реактивным сопротивлением.

Математическая взаимосвязь между напряжением на катушке индуктивности и скоростью изменения тока через нее выглядит следующим образом:

 

Отношение di/dt представляет собой скорость изменения мгновенного тока (i) с течением времени, и измеряется в амперах в секунду. Индуктивность (L) измеряется в Генри, а мгновенное напряжение (u) — в вольтах. Чтобы показать, что происходит с переменным током, давайте проанализируем простую индуктивную схему:

 

Простая индуктивная цепь: ток катушки отстает от напряжения на 90o.

Если мы построим график тока и напряжения для этой простой цепи, то он будет выглядеть примерно так:

 

Как вы помните, изменение напряжения на катушке индуктивности является реакцией на изменение тока, проходящего через нее. Отсюда можно сделать вывод, что мгновенное напряжение равно нулю всякий раз, когда мгновенное значение тока находится в пике (нулевое изменение, или нулевой наклон синусоидальной волны тока), и мгновенное напряжение равно своему пиковому значению всякий раз, когда мгновенный ток находится в точках максимального изменения (точки самого крутого наклона волны тока, в которых она пересекает нулевую линию). Все это приводит к тому, что волна напряжения на 90o не совпадает по фазе с волной тока. На графике видно, как волна напряжения дает «фору» волне тока: напряжение «ведет» ток, а ток «запаздывает» за напряжением.

 

Ели мы на этот график нанесем значения мощности нашей схемы, то все станет еще более интересным:

 

Поскольку мгновенная мощность представляет собой произведение мгновенного напряжения  и мгновенного тока (p = iu), она будет равна нулю, если мгновенное напряжение или ток будут равны нулю. Всякий раз, когда мгновенные значения тока и напряжения имеют положительные значения (выше нулевой линии), мощность так же будет положительна. Аналогично примеру с резистивной цепью, мощность примет положительное значение и в том случае, если мгновенный ток и напряжение будут иметь отрицательные значения (ниже нулевой линии). Однако, вследствие того, что волны напряжения и тока не совпадают по фазе на 90o, бывают случаи, когда ток положителен, а напряжение отрицательно (или наоборот), в результате чего появляются отрицательные значения мгновенной мощности.

Но, что такое отрицательная мощность? Отрицательная мощность означает, что катушка индуктивности отдает энергию обратно в цепь. Положительная же мощность означает, что катушка индуктивности поглощает энергию из цепи. Так как положительные и отрицательные циклы питания равны по величине и продолжительности, в течение полного цикла катушка индуктивности отдает обратно в схему столько же энергии, сколько она потребляет из нее. В практическом смысле это означает, что реактивное сопротивление катушки не рассеивает никакой энергии, чем оно и отличается от сопротивления резистора, рассеивающего энергию в виде тепла. Однако, все вышесказанное справедливо только для идеальных катушек индуктивности, провода которых не имеют никакого сопротивления.

Сопротивление катушки индуктивности, изменяющее силу тока, интерпретируется как сопротивление переменному току в целом, у которого по определению постоянно меняется мгновенная величина и направление. Это сопротивление переменному току похоже на обычное сопротивление, но отличается от него тем, что всегда приводит к фазовому сдвигу между током и напряжением, а так же рассеивает нулевую мощность. Из-за указанных различий, данное сопротивление носит несколько иное название — реактивное сопротивление. Реактивное сопротивление, как и обычное, измеряется в Омах, только обозначается оно символом Х, а не R. Для большей конкретики, реактивное сопротивление катушки индуктивности обычно обозначают заглавной буквой Х с буквой L в качестве индекса: XL.

Поскольку напряжение на катушке индуктивности пропорционально скорости изменения тока, оно будет больше для быстро меняющихся токов, и меньше — для токов с более медленным изменением. Это означает, что реактивное сопротивление любой катушки индуктивности (в Омах) прямопропорционально частоте переменного тока. Точная формула расчета реактивного сопротивления выглядит следующим образом:

 

Если на катушку индуктивностью 10 мГн воздействовать частотами 60, 120 и 2500 Гц, то ее реактивное сопротивление примет следующие значения:

Частота (Гц)Реактивное сопротивление (Ом)
60 3.7699
120 7.5398
2500 157.0796

В уравнении реактивного сопротивления выражение “2πf” имеет важное значение. Оно означает число в радианах в секунду, характеризующее «вращение» переменного тока (один полный цикл переменного тока представляет собой одно полное круговое вращение). Радиан — это единица измерения углов: в одном полном круге есть 2π радиан, точно так же, как в нем есть 360o. Если генератор переменного тока двухполюсный, то он произведет один полный цикл для каждого полного оборота вала, что будет означать 2π радиан или 360o. Если постоянную 2π умножить на частоту в герцах (циклах в секунду), то результатом будет число в радианах в секунду, известное как угловая (циклическая) частота переменного тока.

Помимо выражения 2πf, угловая частота переменного тока может обозначаться строчной греческой буквой ω (Омега). В этом случае формула XL = 2πfL может быть написана как XL = ωL.

Необходимо понимать, что угловая частота является выражением того, насколько быстро проходит полный цикл волны, равный 2π радиан. Она необязательно представляет фактическую скорость вала генератора, производящего переменный ток. Если генератор имеет более двух полюсов, его угловая частота будет кратной скорости вращения вала. По этой причине ω иногда выражается в единицах электрических радиан в секунду, чтобы отличить ее от механического движения.

При любом способе выражения угловой частоты очевидно, что она прямопропорциональна реактивному сопротивлению катушки индуктивности. При увеличении частоты переменного тока (или скорости вращения вала генератора), катушка индуктивности будет оказывать большее сопротивление прохождению тока и наоборот. Переменный ток в простой индуктивной цепи равен напряжению (в Вольтах) поделенному на реактивное сопротивление катушки индуктивности (в Омах). Как видите, это аналогично тому что переменный или постоянный ток в простой резистивной цепи равен напряжению (в Вольтах) поделенному на сопротивление (в Омах). В качестве примера давайте рассмотрим следующую схему:

 

 

Однако, мы должны иметь в виду, что напряжение и ток имеют разные фазы. Как было сказано ранее, напряжение имеет фазовый сдвиг +90o по отношению к току (рисунок ниже). Если представить фазовые углы напряжения и тока математически (в виде комплексных чисел), то мы увидим, что сопротивление катушки индуктивности переменному току обладает следующим фазовым углом:

 

 

Ток на катушке индуктивности отстает от напряжения на 90o.

Математически можно сказать, что фазовый угол сопротивления катушки индуктивности переменному току составляет 90o. Фазовый угол реактивного сопротивления току очень важен при анализе цепей. Особенно эта важность проявляется при анализе сложных цепей переменного тока, где реактивные и простые сопротивления взаимодействуют друг с другом. Он также окажется полезным для представления сопротивления любого компонента электрическому току с точки зрения комплексных чисел (а не скалярных величин сопротивления и реактивного сопротивления).

Цепи индуктивности переменного тока — реактивность и импеданс — индуктивные

Цепи индуктивности переменного тока

Глава 3 — Реакция и импеданс — Индуктивные

Резисторы против индукторов

Индукторы не ведут себя так же, как резисторы. В то время как резисторы просто противодействуют потоку электронов через них (понижая напряжение, прямо пропорциональное току), индукторы противодействуют изменениям тока через них, понижая напряжение, прямо пропорциональное скорости изменения тока. В соответствии с Законом Ленца (о котором вы можете подробнее прочитать здесь) это индуцированное напряжение всегда имеет такую ​​полярность, чтобы попытаться поддерживать ток по его нынешнему значению. То есть, если ток увеличивается по величине, индуцированное напряжение будет «толкать» поток электронов; если ток уменьшается, полярность будет изменяться и «нажимать» на поток электронов, чтобы противостоять уменьшению. Эта оппозиция нынешним изменениям называется реактивностью, а не сопротивлением.

Математически выраженное соотношение между напряжением, падающим на индуктор, и скоростью изменения тока через индуктор, таково:

Переменный ток в простой индуктивной цепи

Выражение di / dt является одним из исчисления, что означает скорость изменения мгновенного тока (i) со временем, в амперах в секунду. Индуктивность (L) находится в Генрисе, и мгновенное напряжение (e), конечно, находится в вольтах. Иногда вы обнаружите скорость мгновенного напряжения, выраженную как «v» вместо «e» (v = L di / dt), но это означает то же самое. Чтобы показать, что происходит с переменным током, давайте проанализируем простую схему индуктора: (рис. Ниже)

Чистая индуктивная схема: ток индуктора отстает от индуктивного напряжения на 90 o .

Если бы мы рассчитали ток и напряжение для этой очень простой схемы, это выглядело бы примерно так: (рис. Ниже)

Чистая индуктивная схема, формы волны.

Помните, что напряжение, падающее на индуктор, является реакцией на изменение тока через него. Поэтому мгновенное напряжение равно нулю всякий раз, когда мгновенный ток находится на пике (изменение нуля или наклон уровня на текущей синусоидальной волне), а мгновенное напряжение находится на пике, где мгновенный ток находится в максимальном изменении (точки самый крутой уклон на текущей волне, где он пересекает нулевую линию). Это приводит к волне напряжения, которая на 90 o не соответствует фазе текущей волны. Глядя на график, волна напряжения, похоже, имеет «начальный старт» на текущей волне; напряжение «приводит» к току, а ток «отстает» от напряжения. (Figurebelow)

Ток задерживает напряжение на 90 o в чистом индуктивном контуре.

Все становится еще интереснее, когда мы закладываем мощность для этой схемы: (рис. Ниже)

В чистой индуктивной цепи мгновенная мощность может быть положительной или отрицательной

Поскольку мгновенная мощность является продуктом мгновенного напряжения и мгновенного тока (p = ie), мощность равна нулю всякий раз, когда мгновенный ток или напряжение равны нулю. Всякий раз, когда мгновенный ток и напряжение являются положительными (выше линии), мощность положительная. Как и в случае с резистором, мощность также положительна, когда мгновенный ток и напряжение отрицательны (ниже линии). Однако из-за того, что волны тока и напряжения 90 o вне фазы, времена когда один положительный, а другой отрицательный, что приводит к столь же частым появлениям отрицательной мгновенной мощности .

Что такое отрицательная сила «индуктивная реактивность»>

Сопротивление индуктора изменению тока переводит на противоположность переменному току в целом, который по определению всегда меняется в мгновенной величине и направлении. Это противодействие переменному току аналогично сопротивлению, но отличается тем, что оно всегда приводит к сдвигу фазы между током и напряжением и рассеивает нулевую мощность. Из-за различий у него другое название: реактивность . Реакция на AC выражается в омах, как и сопротивление, за исключением того, что ее математический символ равен X вместо R. Чтобы быть конкретным, реактивное сопротивление, связанное с индуктором, обычно символизируется прописной буквой X с буквой L в качестве индекса, например это: X L.

Поскольку индукторы понижают напряжение пропорционально скорости изменения тока, они будут падать больше напряжения для более быстро изменяющихся токов и меньше напряжения для медленных изменяющихся токов. Это означает, что реактивное сопротивление в омах для любого индуктора прямо пропорционально частоте переменного тока. Точная формула для определения реактивности следующая:

Если мы разобьем индуктор 10 мГн на частоты 60, 120 и 2500 Гц, он проявит реактивные сопротивления в таблице ниже.

Реакция индуктора 10 мГн:
Частота (Hertz)Реакция (Ом)
603, 7699
1207, 5398
2500157.0796

В уравнении реактивности термин «2πf» (все, что находится справа, кроме L) имеет для себя особый смысл. Это число радианов в секунду, когда переменный ток «вращается», если вы представляете один цикл переменного тока для представления вращения полного круга. Радиан — это единица измерения углов: в одном полном круге есть 2π радиан, так же как 360 o в полном круге. Если генератор переменного тока, создающий переменный ток, представляет собой двухполюсный блок, он будет производить один цикл для каждого полного поворота вала, который составляет каждые 2π радиан или 360 o . Если эта константа 2π умножается на частоту в герцах (циклы в секунду), результатом будет цифра в радианах в секунду, известная как угловая скорость системы переменного тока.

Угловая скорость в системах переменного тока

Угловая скорость может быть представлена ​​выражением 2πf или может быть представлена ​​ее собственным символом, нижней греческой буквой Omega, которая похожа на наш римский нижний регистр «w»: ω. Таким образом, формула реактивности X L = 2πfL также может быть записана как X L = ωL.

Следует понимать, что эта «угловая скорость» является выражением того, насколько быстрыми являются колебания переменного тока, полный цикл равен 2π радианам. Это не обязательно отражает фактическую скорость вала генератора переменного тока. Если генератор переменного тока имеет более двух полюсов, угловая скорость будет кратной скорости вала. По этой причине ω иногда выражается в единицах электрических радианов в секунду, а не (равных) радианах в секунду, чтобы отличить его от механического движения.

В любом случае мы выражаем угловую скорость системы, очевидно, что она прямо пропорциональна реактивному сопротивлению в индукторе. Поскольку частота (или частота вращения генератора) увеличивается в системе переменного тока, индуктор будет оказывать большее сопротивление проходу тока и наоборот. Переменный ток в простой индуктивной цепи равен напряжению (в вольтах), деленному на индуктивное сопротивление (в омах), так как либо переменный, либо постоянный ток в простой резистивной цепи равен напряжению (в вольтах), деленному на сопротивление (в омах). Примерная схема показана здесь: (рис. Ниже)

Индуктивное сопротивление

Фазовые углы

Однако нам нужно иметь в виду, что напряжение и ток здесь не находятся в фазе. Как было показано ранее, напряжение имеет фазовый сдвиг +90 o по отношению к току. (Рис. Ниже). Если представить эти фазовые углы напряжения и тока математически в виде комплексных чисел, мы обнаружим, что сопротивление индуктора течению также имеет фазовый угол:

Ток задерживает напряжение на 90 o в индукторе.

Математически мы говорим, что фазовый угол сопротивления индуктора к току равен 90 o, что означает, что сопротивление индуктора току является положительной мнимой величиной. Этот фазовый угол реактивной оппозиции к току становится критически важным в анализе схемы, особенно для сложных схем переменного тока, в которых взаимодействуют реактивность и сопротивление. Было бы полезно представить противоположность какого-либо компонента течению в терминах комплексных чисел, а не скалярных величин сопротивления и реактивности.

  • ОБЗОР:
  • • Индуктивное сопротивление — это противодействие, которое индуктор предлагает переменному току из-за его фазового сдвига и выделения энергии в своем магнитном поле. Реакция символизируется заглавной буквой «X» и измеряется в омах точно так же, как сопротивление (R).
  • • Индуктивное реактивное сопротивление может быть рассчитано с использованием этой формулы: X L = 2πfL
  • • Угловая скорость цепи переменного тока — это еще один способ выразить свою частоту в единицах электрических радианов в секунду вместо циклов в секунду. Он символизируется строчной греческой буквой «омега» или со.
  • • Индуктивное сопротивление увеличивается с увеличением частоты. Другими словами, чем выше частота, тем больше она противостоит AC-потоку электронов.

Сопротивление цепи переменного тока.

Любая реальная электрическая цепь переменного тока содержит активное сопротивление (сопротивление проводов, приборов, нагрузки), емкостное сопротивление(емкость проводников, конденсаторов) и индуктивное сопротивление (индуктивное сопротивление проводов, катушек, обмотки электродвигателœей и т.д.). Емкостное и индуктивное сопротивления называют реактивным сопротивлением переменному току. Основное отличие реактивного сопротивления от активного состоит по сути в том, что оно влияет на величину тока в цепи, но на нем не происходит превращение электрической энергии в другие виды энергии (к примеру, не выделяется тепло).

Сопротивление конденсатора переменному току называют емкостным сопротивлением XC, оно зависит от частоты переменного тока и от емкости С конденсатора. Сопротивление катушки индуктивности переменному току называют индуктивным сопротивлением ХL, оно зависит от частоты переменного тока и от индуктивности L катушки:

. (14.23)

Полное сопротивление Z цепи, содержащей активное, емкостное и индуктивное сопротивления, соединœенные последовательно, определяется по формуле:

. (14.24)

Закон Ома для такого участка цепи переменного тока имеет вид:

— для амплитудных значений; — для действующих значений тока и напряжения.

Мгновенные значения тока и напряжения на конденсаторе и на катушке индуктивности меняются со сдвигом по фазе, в связи с этим для них закон Ома не записывается.

За более подробным описанием процессов, происходящих в цепи переменного тока, отсылаем к учебнику [2] §§31-35. Там же можно познакомиться с устройством и работой различных генераторов переменного тока, трансформаторов, вопросами производства, передачи и использования электрической энергии [2] §§36-41.

[2]. Г.Я.Мякишев, Б.Б.Буховцев, В.М.Чаругин. Физика. 11 класс. Учебник для общеобразовательных учреждений.- М.: ʼʼПросвещениеʼʼ, 2009 и др.
Размещено на реф.рф
Главы 3-5.

Тема 15. (2 часа)

Механические волны. Длина волны. Скорость волны. Электромагнитные волны. Свойства электромагнитных волн.

Катушка индуктивности в цепи переменного тока. Индуктивное сопротивление | Основы физики сжато и понятно

Для школьников.

В предыдущих статьях рассмотрены цепи переменного тока, содержащие только активное сопротивление и содержащие только емкостное сопротивление.

Сейчас рассмотрим случай, когда в цепи переменного тока находится только катушка индуктивности (индуктивное сопротивление), а активным и емкостным сопротивлениями цепи можно пренебречь.

Начнём с опыта, позволяющего понять, когда появляется и от чего зависит индуктивное сопротивление в цепи переменного тока.

Две маленькие одинаковые электрические лампочки подключались к источникам одинакового напряжения. Но одна (правая) подключалась к источнику постоянного тока (аккумуляторной батарее), а другая (левая) — к источнику переменного тока.

Лампочки светили одинаково, так как количество выделяющегося тепла не зависит от того, какой ток протекает по нитям лампочек (постоянный или переменный).

Затем к лампочкам последовательно подключили катушки индуктивности, сделанные из толстой медной проволоки, содержащей большое число витков. Внутри катушек находятся железные сердечники.

Катушками индуктивности называются катушки, имеющие большую индуктивность и малое активное сопротивление (изготовлены из толстой проволоки). Часто активным сопротивлением такой катушки можно пренебречь.

Опыт показал, что в случае постоянного тока лампочка горит ярко, а в случае переменного тока она светит тускло. Как это объяснить?

В случае постоянного тока лампочка горит ярко, потому что сопротивление катушки мало.

Но почему катушка индуктивности очень сильно ослабляет переменный ток? Продолжаем рассматривать опыт.

Если из катушки (рис. б) постепенно вытягивать железный сердечник, то нить лампочки будет накаляться всё сильнее. При полном вытягивании сердечника лампочка будет светить довольно ярко. Убрав железный сердечник, во много раз уменьшили индуктивность катушки, значит дело в индуктивности.

Так как сила переменного тока быстро меняется, то в катушке возникает ЭДС самоиндукции.

В цепи только с индуктивным сопротивлением приложенное к цепи напряжение в каждый момент времени равно и противоположно ЭДС самоиндукции.

ЭДС самоиндукции по правилу Ленца направлена так, что стремится препятствовать изменению тока, то есть оказывает току сопротивление. Чем больше индуктивность катушки, тем большая ЭДС самоиндукции в ней возникает, тем больше индуктивное сопротивление. Индуктивное сопротивление зависит ещё от частоты тока, чем больше частота, тем больше это сопротивление.

При нарастании тока ЭДС самоиндукции препятствует этому нарастанию, поэтому ток позже достигает максимума, чем в отсутствие самоиндукции.

При убывании тока ЭДС самоиндукции стремится поддерживать ток. Поэтому нулевые значения тока достигаются в более поздний момент, чем в отсутствие самоиндукции.

Таким образом, при наличие индуктивности ток отстаёт по фазе от тока в отсутствие индуктивности, а следовательно, отстаёт по фазе от своего напряжения.

Можно сказать так: из-за явления самоиндукции ток в катушке индуктивности не может меняться скачком и отстаёт от напряжения.

Чем больше частота тока, тем больше будет ЭДС самоиндукции, стремящейся противодействовать изменению тока и тем больше будет сопротивление переменному току.

Индуктивное сопротивление находится по формуле, которая получена теоретически и подтверждена опытом:

Напряжение на индуктивном сопротивлении (индуктивное напряжение) :

Ток в цепи, содержащей только индуктивность, равен отношению напряжения на зажимах источника к индуктивному сопротивлению цепи:

В цепи, содержащей только индуктивное сопротивление, напряжение опережает ток на четверть Т/4 периода (или ток отстаёт от напряжения на четверть периода):

Из рисунка видно, когда ток ещё только проходит через нулевое положение, напряжение уже проходит через максимум, то есть мгновенные значения напряжения и тока меняются согласно уравнениям:

Разность фаз между током и напряжением, равная «пи» пополам, величина отрицательная.

На следующем рисунке показано, как меняются во времени мгновенные значения тока, напряжения и мощности в цепи переменного тока, содержащей только индуктивное сопротивление:

Мгновенные значения мощности находятся через произведение мгновенных значений напряжения и тока, уравнения для которых записаны выше.

Видим, что мгновенная мощность в цепи, обладающей только индуктивностью, изменяется по синусоидальному закону с двойной частотой и имеет равные по модулю положительные и отрицательные значения.

В первую и третью части периода, когда ток в цепи растёт, мощность имеет положительное значение. В эти части периода магнитное поле вокруг электрической цепи усиливается за счёт энергии источника.

Убыванию тока соответствует отрицательное значение мощности. В эти части периода (второй и четвёртый) магнитное поле ослабевает, его энергия идёт обратно к источнику тока.

Таким образом, в цепи, содержащей только катушку индуктивности, происходит периодическая перекачка энергии от источника в энергию магнитного поля катушки индуктивности и возвращение её источнику.

Положительные и отрицательные значения мощности по модулю равны, поэтому средняя мощность за период в такой цепи равна нулю.

Эта мощность не расходуется во внешней части цепи, поэтому называется реактивной индуктивной мощностью:

Она образуется при работе электроприборов (трансформаторов, двигателей, электромагнитов и др., обладающих большой индуктивностью.)

Реактивная мощность в промышленных электрических цепях увеличивает нагрузку на провода и снижает коэффициент мощности цепи, поэтому реактивную нагрузку в электрической цепи на предприятиях снижают (об этом будет сказано в последующих статьях.)

Пример.

Катушка с индуктивностью 0,01 Гн включена в цепь переменного тока напряжением 120 В и частотой 50 Гц. Определить силу тока в катушке, индуктивное напряжение в ней и её реактивную мощность, если активным сопротивлением катушки можно пренебречь.

Используем формулы:

Ответ: 38,2 А; 120 В; 1460 Вт.

Из полученного ответа видим, что всё приложенное к катушке напряжение является индуктивным.

Итак, присутствие индуктивности в цепи переменного тока приводит к сдвигу фаз между током и напряжением в сети (колебания тока отстают от колебаний напряжения).

Емкостная мощность не совершает механической работы, не нагревает проводники.

Катушка индуктивности лишь периодически накапливает энергию магнитного поля и возвращает её обратно источнику тока.

К.В. Рулёва

Предыдущая запись : Конденсатор в цепи переменного тока. Емкостное сопротивление.

Следующая запись :Последовательное соединение сопротивлений. Резонанс напряжений. Мощность переменного тока. Коэффициент мощности.

Ссылки на занятия до электростатики даны в Занятии 1 .

Ссылки на занятия (статьи), начиная с электростатики, даны в конце Занятия 45 .

Ссылки на занятия (статьи), начиная с теплового действия тока, даны в конце Занятия 58 .

Ссылки на занятия, начиная с переменного тока, даны в конце Занятия 70 .

Индуктивность в цепях переменного тока




ЗАДАЧИ:

• обсудить свойства индуктивности в цепи переменного тока.

• обсудить индуктивное реактивное сопротивление.

• вычислить значения индуктивного сопротивления и индуктивности.

• обсудить взаимосвязь напряжения и тока в чисто индуктивном схема.

• уметь вычислять значения для катушек индуктивности, подключенных последовательно или параллельно.

• обсудить реактивную мощность (VAR).

• определить добротность катушки.

ИНДУКТИВНОСТЬ В ЦЕПИ ПЕРЕМЕННОГО ТОКА УСЛОВИЯ

  • ток запаздывает по напряжению — соотношение силы тока и напряжения в чистая индуктивная цепь
  • импеданс (Z) — общий токоограничивающий эффект в цепи переменного тока
  • индуцированное напряжение — напряжение, которое подводится к проводнику при разрезании. линии магнитного потока
  • индуктивность (L) — свойство электрической цепи, при котором напряжение производится резкой магнитных линий
  • индуктивное реактивное сопротивление (XL) — токоограничивающий эффект чистого индуктор
  • Качество
  • (Q) — отношение индуктивного сопротивления к сопротивлению
  • реактивное сопротивление — свойство цепи, ограничивающее ток с помощью кроме сопротивления
  • реактивная мощность (ВАР) — Вольт Ампер реактивная; часто упоминается как безбатная мощность

———————————

В этом модуле обсуждается влияние индуктивности на цепи переменного тока.Устройство также объясняет, как ограничивается ток в индуктивной цепи. поскольку индуктивность влияет на соотношение напряжения и тока.

ИНДУКТИВНОСТЬ

Индуктивность (L) — один из основных типов нагрузок переменного тока. схемы. Некоторое количество индуктивности присутствует во всем переменном токе. цепей из-за постоянно меняющегося магнитного поля (рис. 1). Величина индуктивности одного проводника чрезвычайно мала, и в большинстве случаев это не учитывается в схемных расчетах.

Считается, что цепи содержат индуктивность, когда любой тип нагрузки, содержащей катушку. Для цепей, содержащих катушку, индуктивность учитывается при расчетах схемы. Такие нагрузки, как двигатели, трансформаторы, осветительный балласт и дроссели содержат катушки с проводом.


РИС. 1 Постоянно меняющееся магнитное поле индуцирует напряжение в любой проводник.


РИС. 2 Когда ток течет через катушку, создается магнитное поле. вокруг катушки.


РИС. 3 По мере уменьшения тока магнитное поле схлопывается.

В разделе 10 обсуждалось, что всякий раз, когда ток проходит через катушка с проволокой — вокруг проволоки создается магнитное поле (фиг. 2). Если величина тока уменьшается, магнитное поле схлопывается (фиг. 3). Вспомните из Раздела 10 несколько фактов, касающихся индуктивности:

1. Когда магнитные линии потока проходят через катушку, индуцируется напряжение. в катушке.

2. Индуцированное напряжение всегда противоположно по полярности приложенному напряжению. Это часто называют встречной ЭДС (CEMF).

3. Величина наведенного напряжения пропорциональна скорости изменения. тока.

4. Катушка индуктивности препятствует изменению тока.

Катушки индуктивности на фиг. 2 и 3 подключены к чередующемуся Напряжение. Поэтому магнитное поле непрерывно увеличивается, уменьшается, и меняет полярность.Поскольку магнитное поле постоянно меняет величину и направление, в катушке постоянно индуцируется напряжение. Этот индуцированное напряжение сдвинуто по фазе на 180 ° с приложенным напряжением и всегда против приложенного напряжения (фиг. 4). Поскольку индуцированное напряжение всегда противоположно приложенному напряжению, приложенное напряжение должно преодолеть индуцированное напряжение до того, как ток сможет течь по цепи. Например, предположим, что катушка индуктивности подключена к сети переменного тока напряжением 120 В.Теперь предположим индуктивность индуктора составляет 116 В. Поскольку равная величина приложенного напряжения необходимо использовать для преодоления наведенного напряжения, при этом будет всего 4 В, чтобы протолкнуть ток через сопротивление провода катушки (120 — 116 = 4).


РИС. 4 Приложенное напряжение и индуцированное напряжение сдвинуты по фазе на 180 °. друг с другом.


РИС. 5 Измерение сопротивления катушки.


РИС. 6 Измерение тока цепи с помощью амперметра.

ВЫЧИСЛЕНИЕ ИНДУЦИРОВАННОГО НАПРЯЖЕНИЯ

Величину наведенного напряжения в катушке индуктивности можно вычислить, если сопротивление длины провода в катушке и величина тока в цепи известны. Для Например, предположим, что омметр используется для измерения фактического количества сопротивление в катушке, и обнаружено, что катушка содержит 6 Ом сопротивления провода (Фиг.5).

Теперь предположим, что катушка подключена к цепи переменного тока 120 В и амперметру. измеряет текущий расход 0.2] = 119: 9 В). Обратитесь к векторам в Разделе 13.

ИНДУКТИВНАЯ РЕАКТИВНОСТЬ

Обратите внимание, что индуцированное напряжение может ограничивать протекание тока через схема аналогична сопротивлению. Это индуцированное напряжение равно не сопротивление, но он может ограничить поток тока так же, как сопротивление делает. Это свойство ограничения тока индуктивности называется реактивным сопротивлением. и обозначается буквой X. Это реактивное сопротивление обусловлено индуктивностью, поэтому оно называется индуктивным реактивным сопротивлением и обозначается XL, произносится как «X». sub L.»Индуктивное реактивное сопротивление измеряется в омах так же, как сопротивление есть и может быть вычислено, когда значения индуктивности и частоты равны известный. Для определения индуктивного сопротивления можно использовать следующую формулу.

XL = 2πFL

где:

XL = индуктивное реактивное сопротивление

2 = постоянная

пи или π = 3,1416

F = частота в герцах (Гц)

L = индуктивность в генри (Гн)

Индуктивное реактивное сопротивление — это индуцированное напряжение и, следовательно, пропорционально к трем факторам, определяющим индуцированное напряжение:

1.Количество витков провода

2. Напряженность магнитного поля

3. Скорость режущего действия (относительное движение индуктора и магнитные линии потока)


РИС. 7 Катушки с более близкими витками создают большую индуктивность, чем катушки с далеко друг от друга витками.

Определяются количество витков провода и напряженность магнитного поля. по физической конструкции индуктора. Такие факторы, как размер используемого провода, количество витков, насколько близко друг к другу витки, и тип материала сердечника определяют величину индуктивности (в Генри, H) катушки (РИС.7). Скорость резания пропорциональна к частоте (Гц). Увеличение частоты вызовет магнитное линии потока, чтобы сократить проводники с большей скоростью, и, таким образом, будет производить более высокое наведенное напряжение или большее индуктивное сопротивление.


РИС. 9 Условные обозначения катушек индуктивности.

СХЕМАТИЧЕСКИЕ СИМВОЛЫ

На схематическом изображении индуктора изображена катушка с проводом. Несколько символов для катушек индуктивности показаны на фиг.9. Показанные символы с двумя параллельными линиями представляют индукторы с железным сердечником, а символы без параллельных линий обозначены индукторы с воздушным сердечником.

ИНДУКТОРЫ, ПОДСОЕДИНЕННЫЕ СЕРИИ

При последовательном включении катушек индуктивности (РИС. 10) общая индуктивность цепи (LT) равна сумме индуктивностей всех катушек индуктивности.

LT = L1 + L2 + L3

Суммарное индуктивное сопротивление (XLT) последовательно соединенных катушек индуктивности равно сумма индуктивных сопротивлений всех катушек индуктивности.

XLT = XL1 + XL2 + XL3

ИНДУКТОРЫ, СОЕДИНЕННЫЕ ПАРАЛЛЕЛЬНО

При параллельном подключении катушек индуктивности (РИС. 11) общая индуктивность можно найти аналогично нахождению полного сопротивления параллельного схема. Обратное значение полной индуктивности равно сумме обратные величины всех катушек индуктивности.


РИС. 10 последовательно соединенных индукторов.


РИС. 11 катушек индуктивности подключены параллельно.

Формулу произведения на сумму можно также использовать для определения общей индуктивности. параллельных индукторов.

LT = L1 x L2 / L1 + L2

Если значения всех индукторов одинаковы, общая индуктивность может можно найти, разделив индуктивность одной катушки индуктивности на общее количество индукторов.

LT = L / N

По аналогичным формулам можно найти полное индуктивное сопротивление индукторы соединены параллельно.

1 / XLT = 1 / XL1 + 1 / XL2 + 1 / XL3

ОТНОШЕНИЯ НАПРЯЖЕНИЯ И ТОКА В ИНДУКТИВНОЙ ЦЕПИ


РИС. 12 Наведенное напряжение пропорционально скорости изменения Текущий.


РИС. 13 Никакое напряжение не индуцируется, когда ток не изменяется.


РИС. 14 Ток отстает от приложенного напряжения на 90 °.

В разделе 11 обсуждалось, что когда ток течет через чистую резистивная цепь, ток и напряжение находятся в фазе друг с другом.В чисто индуктивной цепи ток отстает от напряжения на 90 °. Вначале это может показаться невыполнимым условием, пока отношение применяемых учитывается напряжение и наведенное напряжение. Как нынешний и применяемый Напряжение может сдвинуться по фазе на 90 ° друг с другом. сравнивая соотношение тока и наведенного напряжения (фиг. 12). Напомним, что индуцированное напряжение пропорционально скорости изменение силы тока (скорости режущего действия).В начале осциллограммы, ток отображается с максимальным значением в отрицательном направлении. В это время ток не меняется, поэтому индуцированное напряжение равно нулю. Когда сила тока начинает уменьшаться, магнитное поле создает потоком ток уменьшается или коллапсирует и начинает индуцировать напряжение в катушку, когда она прорезает проводники (РИС. 13).

Наибольшая скорость изменения тока происходит, когда ток проходит от отрицательный, через ноль, и начинает увеличиваться в положительном направлении (ИНЖИР.13). Поскольку ток изменяется с наибольшей скоростью, индуцированная напряжение максимальное.

Когда ток приближается к своему пиковому значению в положительном направлении, скорость изменения уменьшается, вызывая уменьшение наведенного напряжения. Индуцированная напряжение снова будет равно нулю, когда ток достигнет своего пикового значения и магнитное поле перестает расширяться.

Видно, что ток, протекающий через катушку индуктивности, является ведущим индуцированное напряжение на 90 °.

Поскольку индуцированное напряжение сдвинуто по фазе на 180 ° с приложенным напряжением, ток будет отставать от приложенного напряжения на 90 ° (рис. 14).

МОЩНОСТЬ В ИНДУКТИВНОЙ ЦЕПИ

В чисто резистивной цепи истинная мощность или ватт равна произведение напряжения и тока.

Однако в чисто индуктивной цепи не вырабатывается истинная мощность или ватт. Напомним, что напряжение и ток должны быть либо положительными, либо отрицательными. прежде, чем будет произведена настоящая сила.Поскольку напряжение и ток равны 90 ° не совпадают по фазе друг с другом в чистой индуктивной цепи, ток и напряжение будет иметь разную полярность в 50% случаев и в одно и то же время. полярность в 50% случаев. В течение периода времени, когда текущая и напряжения имеют одинаковую полярность, питание на цепь подается в форма создания магнитного поля. Когда ток и напряжение равны противоположной полярности, мощность возвращается в цепь в качестве магнитного поле схлопывается и снова индуцирует напряжение в цепи.2 R потери, потери на вихревые токи и гистерезис убытки.

РИС. 15 Соотношение напряжения и тока в разных частях цикла.

Форма волны тока и напряжения на фиг. 15 было разделено на четыре секции:

A, B, C и D. В течение первого периода времени, обозначенного A, текущий отрицательный, а напряжение положительное. В этот период энергия передается в цепь при схлопывании магнитного поля.В течение второй период времени, раздел B, и напряжение, и ток положительны. Энергия используется для создания магнитного поля. В третьем временном периоде C, ток положительный, а напряжение отрицательное. Сила снова возвращается в схему, когда поле схлопывается. Во время четвертого период времени D, как напряжение, так и ток отрицательны. Сила снова используется для создания магнитного поля. Если количество энергии, используемой для магнитное поле вычитается из возвращаемой мощности, результат будет нулевым.

РЕАКТИВНАЯ МОЩНОСТЬ

Хотя, по существу, истинная мощность не используется, за исключением ранее упомянутых потерь, электрическое измерение, называемое VAR, используется для измерения реактивная мощность в чисто индуктивной цепи. VARs — это аббревиатура для вольт-ампер-реактивных. VAR можно вычислить как ватт, за исключением индуктивного значения заменяются на значения сопротивления в формулах.

ВАр равно количеству тока, протекающего через индуктивную цепь. умноженное на напряжение, приложенное к индуктивной части цепи.2 L x XL

где

EL = напряжение, приложенное к катушке индуктивности

IL = текущий расход через индуктор

XL = индуктивное реактивное сопротивление

Q ИНДУКТОРА


РИС. 16 катушек индуктивности содержат внутреннее сопротивление.


РИС. 17 Добротность катушки индуктивности — это отношение индуктивного реактивного сопротивления как по сравнению с сопротивлением. Буква Q означает качество.

До сих пор в этом разделе обычно предполагалось, что индуктор не имеет сопротивления, и это индуктивное реактивное сопротивление является единственным ограничивающим ток фактор.На самом деле это не так. Поскольку индукторы на самом деле являются катушками провода, все они содержат некоторое количество внутреннего сопротивления. Индукторы на самом деле выглядит как катушка, соединенная последовательно с некоторым сопротивлением (РИС. 16). Величина сопротивления по сравнению с индуктивным сопротивлением определяет добротность катушки. Буква Q означает качество.

Катушки индуктивности с более высоким отношением индуктивного реактивного сопротивления к сопротивлению считаются индукторами более высокого качества.Индуктор построен с большим проводом будет иметь низкое сопротивление провода и, следовательно, более высокое Q (фиг.17). Индукторы, состоящие из множества витков небольшого провода, имеют гораздо более высокое сопротивление и, следовательно, более низкое значение Q.

.

Чтобы определить добротность катушки индуктивности, разделите индуктивное реактивное сопротивление на сопротивление.

Q = XL / R


РИС. 18 Импеданс катушки представляет собой комбинацию сопротивления провода и индуктивного сопротивления. реактивное сопротивление.

Хотя индукторы имеют некоторое сопротивление, индукторы с Q = 10 или больше обычно считаются чистыми индукторами.Один раз отношение индуктивного реактивного сопротивления становится в 10 раз больше сопротивления, величина сопротивления считается незначительной. Например, предположим индуктор имеет индуктивное сопротивление 100 Ом и сопротивление провода 10 Ом. Индуктивная реактивная составляющая в цепи находится на 90 ° вне фаза с резистивной составляющей. Эти отношения создают право треугольник (фиг.18). Общий ток ограничивающий эффект катушки индуктивности представляет собой комбинацию индуктивного реактивного сопротивления и сопротивления.Эта сумма токоограничивающий эффект называется импедансом и обозначается буквой Z. Импеданс цепи представлен гипотенузой прямоугольный треугольник, образованный индуктивным сопротивлением и сопротивлением. К вычислить значение импеданса для катушки, индуктивного реактивного сопротивления и сопротивление должно быть добавлено. Поскольку эти два компонента образуют ножки прямоугольный треугольник и импеданс образует гипотенузу, сложение вектора должны быть заняты.

Обратите внимание, что значение полного сопротивления катушки индуктивности составляет всего 0,5. O больше, чем значение индуктивного реактивного сопротивления.

РЕЗЮМЕ

• Наведенное напряжение пропорционально скорости изменения тока.

• Индуцированное напряжение всегда противоположно полярности приложенного напряжения.

• Индуктивное реактивное сопротивление — это противодействующее напряжение, ограничивающее ток, как и сопротивление.

• Индуктивное реактивное сопротивление измеряется в омах.

• Индуктивное реактивное сопротивление пропорционально индуктивности катушки и частота линии.

• Индуктивное реактивное сопротивление обозначается XL.

• Индуктивность измеряется в генри (H) и обозначается буквой. Л.

• При последовательном подключении катушек индуктивности общая индуктивность равна сумме всех индукторов.

• При параллельном подключении катушек индуктивности обратная величина индуктивность равна сумме обратных величин всех катушек индуктивности.

• Ток отстает от приложенного напряжения на 90 ° в чисто индуктивной цепи.

• Все катушки индуктивности содержат некоторое сопротивление.

• Добротность катушки индуктивности — это отношение индуктивного реактивного сопротивления к сопротивление.

• Катушки индуктивности с добротностью 10 обычно считаются «чистыми» индукторами.

• Чисто индуктивные цепи не содержат истинной мощности или ватт.

• Реактивная мощность измеряется в барах.

• VARs — это сокращение от вольт-ампер-реактивного.

ВИКТОРИНА

1. На сколько градусов не совпадают по фазе ток и напряжение с каждым другой в чисто резистивной цепи?

2. На сколько градусов не совпадают по фазе ток и напряжение с каждым другой в чисто индуктивной цепи?

3. Чему пропорционально индуктивное сопротивление?

4. Четыре катушки индуктивности, каждая с индуктивностью 0.6 H, подключены в серии. Какая общая индуктивность цепи?

5. Три индуктора подключены параллельно. Индуктор 1 имеет индуктивность 0,06 H; индуктор 2 имеет индуктивность 0,05 Гн; а индуктор 3 имеет индуктивность 0,1 ч. Какова общая индуктивность этой цепи?

6. Если три катушки индуктивности, о которых идет речь 5, были соединены последовательно, что будет индуктивное сопротивление цепи? Предположим, что катушки индуктивности подключен к линии 60 Гц.

7. Катушка индуктивности подключена к линии 240 В, 1000 Гц. Ток в цепи составляет 0,6 А. Какова индуктивность катушки индуктивности?

8. Катушка индуктивности 3,6 Гн подключена к сети 480 В, 60 Гц. линия. Сколько тока будет протекать в этой цепи?

9. Если частота в вопросе 8 снижена до 50 Гц, какой ток будет течь в контуре?

10. Катушка индуктивности имеет индуктивное сопротивление 250 Ом при подключении к линия 60 Гц.Каким будет индуктивное сопротивление, если катушка индуктивности подключена на линию 400 Гц?

РЕАЛЬНЫЕ ПРИЛОЖЕНИЯ ДЛЯ МИРА

1. Вы работаете электриком по установке люминесцентных ламп. Ты обратите внимание, что фары были произведены в Европе и что балласты рассчитаны на для работы в сети 50 Гц. Будут ли эти балласты повреждены более чем ток, если они подключены к 60 Гц? Если есть проблема с этими огни, что будет наиболее вероятной причиной неисправности?

2.У вас есть задача заказать замену катушки индуктивности для той, у которой есть стать неисправным. Информация на заводской табличке закрашена и не может быть прочитан. Машина, в которой находится индуктор, работает от 480 В при частоте 60 Гц. Другая машина имеет идентичный индуктор в нем, но закрашен и его шильдик. Токоизмерительные клещи показывает ток 18 А, а вольтметр показывает падение напряжения 324 В на катушке индуктивности машины, которая все еще работает.После выключив питание и заблокировав панель, вы отключаете индуктор в работающей машине и измерьте сопротивление провода 1,2 Ом с омметр. Использование идентичного индуктора в действующей машине в качестве пример, какое значение индуктивности следует заказать, и какое будет минимальный рейтинг VAR индуктора? Если вас беспокоит сумма сопротивления провода в индукторе при заказе? Поясните свой ответ.

Разница между сопротивлением переменного и постоянного тока

Разница между сопротивлениями переменного и постоянного тока и как ее рассчитать?

Сопротивление

Свойство вещества или материала, которое препятствует прохождению электрического тока через него, называется сопротивлением ИЛИ,

Сопротивление — это способность цепи или элемента (который называется резистором) противодействовать прохождение тока через него.

Примеры резисторов со способностью иметь высокое сопротивление: дерево, воздух, слюда, стекло, резина, вольфрам и т.д. по « R ».

Сопротивление переменного тока

Проще говоря, сопротивление в цепях переменного тока называется импедансом. Или

Общее сопротивление (сопротивление, индуктивное реактивное сопротивление и емкостное реактивное сопротивление) в цепях переменного тока называется импедансом (Z).

Пояснение:

Когда переменный ток проходит через провод (резистор, катушка индуктивности, конденсатор), ток создает магнитное поле на этом проводе, которое противодействует протеканию переменного тока в нем вместе с сопротивлением этого провода. Эта противоположная причина называется индуктивностью, или индуктивность — это свойство катушки (или провода), которое противодействует любому увеличению или уменьшению тока или потока через нее. Кроме того, мы знаем, что индуктивность существует только в переменном токе, потому что величина тока, непрерывно меняющаяся

Индуктивное реактивное сопротивление X L , является свойством катушки или провода в цепи переменного тока, которое противодействует изменению тока.Единица индуктивного реактивного сопротивления такая же, как и сопротивление, емкостное реактивное сопротивление, т. Е. Ом (Ом), но характерным символом емкостного реактивного сопротивления является X L .

Аналогично,

Емкостное реактивное сопротивление в емкостной цепи является противодействием протеканию тока только в цепях переменного тока. Единица емкостного реактивного сопротивления такая же, как и сопротивление, индуктивное реактивное сопротивление, т. Е. Ом (Ом), но типичный символ емкостного реактивного сопротивления — X C .

Измерение сопротивления переменному току

Формулы электрического сопротивления и импеданса в цепях переменного тока

В цепях переменного тока (емкостная или индуктивная нагрузка), сопротивление = импеданс i.е., R = Z

Z = √ (R 2 + X L 2 )… В случае индуктивной нагрузки

Z = √ (R 2 + X C 2 ) … В случае емкостной нагрузки

Z = √ (R 2 + (X L — X C ) 2 … в случае как индуктивной, так и емкостной нагрузки.

Полезно знать:

Где ;

X L = Индуктивное реактивное сопротивление

X L = 2π f L… Где L = Индуктивность в Генри

А;

X C = Емкостное реактивное сопротивление

X C C / 2π f C… Где C = емкость в фарадах.

Сопротивление постоянному току

Мы знаем, что в цепях постоянного тока нет концепции индуктивных и емкостных сопротивлений. т.е. емкостные и индуктивные реактивные сопротивления в цепях постоянного тока равны нулю, потому что в цепях постоянного тока нет частоты, то есть величина постоянного тока постоянна. Следовательно, в игру вступает только исходное сопротивление провода.

Полезно знать:

Вот почему сопротивление, обеспечиваемое проводом, для постоянного тока ниже, чем для переменного, так как линии переменного тока нуждаются в большей изоляции, чем постоянного тока.

Измерение сопротивления постоянному току

Формулы электрического сопротивления

В цепях постоянного тока сопротивление рассчитывается по закону Ома.

R = V / I.

Полезно знать:

При решении электрических цепей для определения сопротивления, и вы не уверены, какое из них следует учитывать, сопротивление переменному или постоянному току, тогда, если проходящий ток является переменным, тогда принимайте сопротивление переменного тока, иначе, если ток пройдено — это постоянный ток, возьмите сопротивление постоянному току.

Что больше — сопротивление постоянному или переменному току?

Поскольку мы знаем, что частота в источнике постоянного тока равна нулю, поэтому отсутствует скин-эффект (поведение переменного тока, протекающего через поверхность i.е. внешний слой проводника вместо сердечника провода). в цепях постоянного тока. Из-за скин-эффекта сопротивление переменного тока больше в цепях переменного тока, чем питание постоянного тока в цепях постоянного тока .

Формула скин-эффекта

δ = √ (2ρ / ωµ)

Где;

  • δ = Глубина скин-эффекта
  • ρ = Удельное сопротивление
  • ω = 2π f = Угловая частота
  • µ = Проницаемость проводника

Короче говоря, частота прямо пропорциональна скин-эффекту i.е. если частота увеличивается, скин-эффект также увеличивается там, где нет частотного и скин-эффекта в DC.

Как правило;

Сопротивление переменного тока = 1,6 x Сопротивление постоянному току

Связанные сообщения:

Цепь переменного тока, содержащая сопротивление и индуктивность | Текущий

Ток, протекающий через постоянный ток. Схема задается напряжением, приложенным к цепи, деленным на сопротивление цепи. Но в переменном токе цепи эти простые отношения не выдерживают.Переменный ток создает магнитные эффекты и переменные ЭДС. создавать электростатические эффекты, и эти эффекты необходимо учитывать. При сравнительно низком напряжении и большом токе магнитные эффекты больше, а электростатические эффекты незначительны, но при высоком напряжении электростатические эффекты заметны.

Цепи переменного тока только с сопротивлением:

переменного тока Схема, содержащая безындуктивное сопротивление R Ом, показана на рис. 24 (а).Когда ток течет по цепи, падение напряжения на сопротивлении равно произведению тока на сопротивление цепи, то есть

.

В = ИК вольт,

, где V — напряжение, приложенное к цепи, а I — ток в амперах, протекающий по цепи.

Когда схема содержит только сопротивление, приложенное напряжение и ток схемы находятся в фазе, то есть векторы тока и напряжения всегда работают в одном направлении, а угол сдвига фаз между ними равен нулю.Это показано на рис. 24 (б) и 24 (в).

Мощность, потребляемая цепью:

Мощность, потребляемая постоянным током. Схема в ваттах определяется как произведение напряжения, приложенного к цепи, и тока в амперах, протекающего по цепи. В переменном токе В схеме это соотношение сохраняется только для мгновенных значений. На практике средняя мощность в ваттах, потребляемая переменным током. схема за период Учитывается. Это значение показывает ваттметр, подключенный к цепи.

Средняя мощность, потребляемая переменным током. Схема задается как произведение трех факторов —r.m.s. напряжение, приложенное к цепи, среднеквадратичное. ток, протекающий по цепи, и коэффициент мощности цепи.

P = VI cos θ ватт,

где,

P = средняя мощность, потребляемая цепью за период,

В = среднеквадратичное значение напряжение, приложенное к цепи,

I = среднеквадратичное значение ток, протекающий по цепи, и

θ = фазовый угол между V и I.

Для переменного тока цепь, содержащая только неиндуктивное сопротивление (омическое сопротивление), θ = 0 ° и cos θ = 1.

. . . P = VI ватт.

Снова V = IR вольт.

. . . P = IR x I = I 2 R Вт.

Таким образом, переменный ток цепь, содержащая сопротивление, ведет себя только как постоянный ток. схема. Мощность синусоидальная, и ее частота вдвое больше, чем у напряжения или тока.

А.C. Цепи, содержащие только индуктивность:

Когда переменный ток Схема содержит катушку, в ней есть индуктивность. Наличие индуктивности в цепи приводит к отставанию тока от приложенного напряжения. Если цепь имеет только индуктивность (и незначительное сопротивление), ток отстает от напряжения на 90 ° или напряжение опережает ток на 90 °. Это показано на рис. 25.

Пусть переменный ток Схема содержит пренебрежимо малое сопротивление и индуктивность в Генри.Это показано на рис. 25 (а). Поскольку ток отстает от приложенного напряжения на 90 °, ток в цепи равен нулю, когда напряжение максимальное, и ток максимален, когда напряжение равно нулю. Это показано на волновой диаграмме на рис. 25 (б).

Как только напряжение подается на цепь, ток не течет по цепи. Ток в цепи начинает постепенно расти после того, как напряжение достигает максимального значения. Точно так же, когда цепь выключена, на ее выводах нет напряжения.Но ток по-прежнему остается в цепи и постепенно гаснет.

Из-за изменения тока э.д.с. самоиндукции устанавливается в катушке. Это вызвало ЭДС. действует против приложенного напряжения, т. е. приложенное напряжение нейтрализует самоиндуцированную ЭДС. в каждое мгновение. Волна самоиндукции ЭДС. показан пунктирной линией на волновой диаграмме. Векторная диаграмма, представляющая векторы напряжения и тока, показана на рис. 25 (с).

Индуктивность в сочетании с частотой питания противодействует прохождению тока через катушку.Это сопротивление известно как индуктивное реактивное сопротивление. Обычно обозначается X L и выражается в омах. Если катушка имеет индуктивность L Генри и если частота питающей сети составляет f герц, индуктивное сопротивление

X L = 2πfL Ом.

Напряжение, приложенное к катушке, деленное на индуктивное реактивное сопротивление, дает ток, протекающий через катушку. Если приложенное напряжение составляет В вольт, а ток через катушку — 1 ампер, то

I = V / X L ампер,

или V = I x X L вольт.

Мощность, потребляемая цепью:

Среднее значение мощности, потребляемой индуктивностью за период, равно нулю. Поскольку ток отстает от напряжения на 90 °, фазовый угол между напряжением и током, то есть θ = 90 °. Следовательно, коэффициент мощности cosθ = cos 90 ° = 0.

Отсюда средняя мощность, потребляемая катушкой:

P = VI cos θ = 0.

Для четверти цикла, когда ток в цепи постепенно увеличивается, магнитное поле внутри и вокруг катушки увеличивается.В это время индуктивность потребляет энергию от источника и сохраняет ее в магнитном поле. Но в течение следующей четверти цикла, когда ток постепенно уменьшается, магнитное поле схлопывается вместе с током, и накопленная энергия возвращается обратно в источник питания. Следовательно, средняя мощность равна нулю.

Поскольку индуктивность не поглощает энергию, она не может выполнять какую-либо полезную работу в цепи. Вся полезная работа выполняется сопротивлением.

Пр. 1:

Найдите ток, который будет протекать через катушку с незначительным сопротивлением и индуктивностью 0.02 генри при подключении к источнику питания 100 вольт, 50 герц. Какой будет ток при удвоении частоты?

Раствор:

Здесь L = 0,02 Генри, V = 100 В и f = 50 Гц.

Индуктивное реактивное сопротивление,

X L = 2πfL = 2 x 3,14 x 50 x 0,02 = 6,28 Ом.

Ток, протекающий по цепи,

I = V / X L = 100 / 6,28 = 15,92 ампер.

При удвоении частоты f = 100 герц.

. . . X L = 2 x 3,14 x 100 x 0,02 = 12,56 Ом.

Ток, протекающий по цепи,

I = 100 / 12,56 = 7,96 ампер

Последовательные цепи переменного тока, содержащие сопротивление и индуктивность:

Нет переменного тока схема содержит только индуктивность. На практике индуктивность всегда связана с определенным сопротивлением. Некоторые устройства, такие как дроссельная катушка или дроссельная катушка, имеют небольшое сопротивление и сравнительно большую индуктивность.Но ни одна катушка не имеет индуктивности без сопротивления.

Сопротивление и индуктивность не существуют как два отдельных элемента в цепи. Они остаются связанными вместе. Но для изучения они показаны отдельно. Поскольку один и тот же ток течет через сопротивление и индуктивность в каждый момент времени и поскольку сумма падений напряжения в сопротивлении и индуктивности дает общее приложенное напряжение на катушке, эти два элемента считаются соединенными последовательно. Таким образом, индуктивную катушку можно рассматривать как состоящую из двух a.c. цепи, соединенные последовательно, одна из которых содержит только сопротивление, а другая — только индуктивность.

переменного тока Схема, содержащая сопротивление R Ом последовательно с индуктивностью Генри, показана на рис. 26 (а). Пусть,

I = среднеквадратичное значение значение тока, протекающего по цепи,

В R = среднеквадратичное значение падение напряжения на сопротивлении

= ИК-напряжение в фазе с I,

В L = среднеквадратичное значение падение напряжения на индуктивности

= IX L вольт, опережающий I на 90 °, и

В = р.РС. напряжение, приложенное ко всей цепи, представляет собой векторную сумму V R и V L .

Векторная диаграмма схемы представлена ​​на рис. 26 (с). Из этой диаграммы мы находим, что ток отстает от приложенного напряжения на угол θ (или приложенное напряжение опережает ток цепи на угол θ), так что tan θ = X L / R. Это также показано на волновой диаграмме на рис. 26 (б).

На векторной диаграмме V R , V L и V образовали прямоугольный треугольник, в котором V R представляет собой прилегающую сторону, V L представляет противоположную сторону или перпендикуляр, а V представляет гипотенузу. .

√R 2 + X L 2 — это противодействие протеканию тока, обеспечиваемое совместно сопротивлением и реактивным сопротивлением цепи. Это сопротивление известно как сопротивление цепи. Обычно обозначается буквой Z и выражается в омах. Таким образом,

Мощность, потребляемая цепью:

Поскольку индуктивность не поглощает энергию, в цепи только сопротивление поглощает энергию. Опять же, мощность, поглощаемая сопротивлением, равна произведению напряжения на сопротивлении и тока, протекающего через него.Следовательно, мощность, потребляемая всей схемой, равна —

.

P = V R I Вт.

Также V R = IR вольт,

. . . P = IR x I = I 2 R Вт.

Из векторной диаграммы:

cosθ = В R / В

или, V R = Vcos θ вольт.

. . . P = Vcos θ x I = VI cos θ ватт.

cos θ — коэффициент мощности цепи.В этой схеме, поскольку ток отстает от приложенного напряжения, коэффициент мощности схемы представляет собой коэффициент мощности с запаздыванием, а фазовый угол θ называется углом запаздывания.

Коэффициент мощности cos θ = V R / V = ​​IR / IZ = R / Z запаздывание.

Индуктивно-реактивные и фазовые диаграммы

Предположим, что индуктор подключен непосредственно к источнику переменного напряжения, как показано на рис. Разумно предположить, что сопротивление пренебрежимо мало, потому что на практике мы можем сделать сопротивление катушки индуктивности настолько малым, что оно окажет незначительное влияние на схему.График показывает напряжение и ток как функции времени. (б) начинается с максимального напряжения. Обратите внимание, что ток начинается с нуля, затем повышается до своего пика после управляющего им напряжения (как показано в предыдущем разделе, когда было включено напряжение постоянного тока).

Источник переменного напряжения последовательно с индуктором

(a) Источник переменного напряжения, включенный последовательно с индуктором, имеющим незначительное сопротивление. (б) График зависимости тока и напряжения на катушке индуктивности от времени.

Когда напряжение становится отрицательным в точке а, ток начинает уменьшаться; оно становится нулевым в точке b, где напряжение является самым отрицательным. Затем ток становится отрицательным, снова вслед за напряжением. Напряжение становится положительным в точке c, где оно начинает делать ток менее отрицательным. В точке d ток проходит через ноль, когда напряжение достигает своего положительного пика, чтобы начать следующий цикл. Следовательно, когда на катушку индуктивности подается синусоидальное напряжение, оно опережает ток на одну четверть цикла или на фазовый угол 90º.

Ток отстает от напряжения, так как катушки индуктивности препятствуют изменению тока. Изменение тока вызывает ЭДС. Это считается эффективным сопротивлением катушки индуктивности переменному току. Среднеквадратичный ток I RMS через катушку индуктивности L определяется версией закона Ома: $ I_ {rms} = \ frac {V_ {rms}} {X_L} $, где V rms — среднеквадратичное напряжение на катушке индуктивности. и $ X_L = 2 \ pi \ nu L $, где $ \ nu $ — частота источника переменного напряжения в герцах. X L называется индуктивным реактивным сопротивлением.Поскольку катушка индуктивности препятствует прохождению тока, X L имеет единицы измерения Ом (1 Гн = 1 Ом · с, так что частота, умноженная на индуктивность, имеет единицы (циклов / с) (Ом · с) = Ом), что соответствует его роли в качестве эффективное сопротивление.

Представление фазора

Напряжение на катушке индуктивности «ведет» ток в соответствии с законом Ленца. Следовательно, вектор, представляющий ток и напряжение, будет иметь вид. Опять же, вектора — это векторы, вращающиеся против часовой стрелки с частотой $ \ nu $ (вы можете видеть, что напряжение опережает ток).В последующих выпусках Atoms будет обсуждаться, как эти векторы можно использовать для анализа цепей RC, RL, LC и RLC.

Векторная диаграмма

Векторная диаграмма для цепи переменного тока с индуктором.

(PDF) СОПРОТИВЛЕНИЕ ПЕРЕМЕННОГО ТОКА И ИНДУКТИВНОСТЬ РЕЛЬСОВ

Payne: Сопротивление переменному току и индуктивность рельсов: Выпуск 2

Рельсы

(см. Фото выше). Гусеница была поднята над измерительным столом на 60 мм, чтобы ее влияние

было минимальным.

Индуктивность была измерена в диапазоне частот от 2 МГц до 8 МГц, выбранном для обеспечения того, чтобы полное сопротивление

было не ниже 10 Ом, поскольку измерения не так точны ниже этого уровня и не слишком

близко к самому себе. резонансная частота, измеренная как 60 МГц (Приложение 7). Измеренная индуктивность составила

с поправкой на соединительные провода, SRF и погрешность калибровки индуктивности 1%, а среднее значение для

в этом диапазоне составило 1.18 мкГн.

Гровер (ссылка 17 p 39) дает индуктивность этой обратной цепи с круглыми проводниками как:

L = µo2ℓ / (2π) [Ln 2 D / d + µr / 4- d / ℓT] A3.1.1

где ℓ — длина каждого из двух проводников

D — расстояние между центрами проводников

d — диаметр проводника

мкм — относительная проницаемость проводника

Модель рельсов, вероятно, изготовлена из нейзильбера, так что относительная проницаемость равна единице (µr = 1)

подтверждается тем фактом, что они не были притянуты к мощному магниту.

Если диаметр d выбран так, чтобы площадь поперечного сечения была такой же, как у рельса A, тогда d = (4A / π) 0,5. Для этого нам нужно знать площадь рельса

, но это трудно определить на рельсе малой модели, но площадь рельса полного размера

известна, поэтому площадь рельса модели будет меньше на куб масштабирования. фактор. Размер рельса модели

составляет от 3,5 мм до 1 фута, то есть 1:87, а площадь полноразмерного рельса UIC 60 составляет 76,70 см2, так что

модели составляет 76.70 10-4 / (87) 3 109 = 11,65 мм 2, а диаметр, который дает такую ​​же площадь, составляет 3,85 мм.

Использование этого диаметра Уравнение A3.1 дает индуктивность 0,64 мкГн по сравнению с измеренным значением 1,1

мкГн, то есть погрешность более 40%.

Таким образом, использование эквивалентного диаметра дает очень плохую оценку индуктивности шины.

Альтернативный эквивалент — прямоугольная рейка с такой же площадью поперечного сечения и такой же периферией.

Для обратной цепи из двух прямоугольных параллельных стержней длиной ℓ, шириной w, толщиной t и расстоянием между центрами D

индуктивность определяется Терманом как (ссылка 5, p52):

L = µo2ℓ / (2π) [ Ln D / (w + t) +1.5 –D / ℓ + 0,2235 (w + t) / ℓ] A3.1.2

Вышеприведенное уравнение предназначено для проводника с относительной магнитной проницаемостью, равной единице, поэтому в индуктивность

учитывается вклад поля внутри провода. в коэффициенте 1,5. Для круглого проводника вклад

равен, т. Е. Коэффициент 1,5 равен 1,25 +1/4. Для прямоугольного проводника коэффициент, вероятно,

ближе к 1,4 (то есть 1,35 + 1 / 6,5) на постоянном токе (см. Раздел 4.3), и он дополнительно уменьшается в измерениях

из-за скин-эффекта (толщина скин-слоя была 0.12 мм при 5 МГц).

Для рельса UIC 60 A = 76,7 см2 и периферия 683 мм, поэтому из уравнения 3.1 w = 317 мм и t = 24

мм, что при масштабировании на 87 дает ширину модели w = 3,65 мм и t = 0,28 мм.

Использование этих размеров в приведенном выше уравнении дает индуктивность, которая на 5,5% ниже измеренного значения

. Это можно исправить, увеличив член с 1,5 до 1,7.

Итак, уравнение, которое лучше всего описывает индуктивность модельной шины:

LE = µoℓT / (2π) [Ln D / (w + t) +1.7] Генри A3.1.3

[Когда размеры и расстояние между рельсами малы по сравнению с длиной, как здесь, последние два фактора в

уравнении A3.1.2. увеличили индуктивность только на 0,5%, поэтому им можно пренебречь] .

Это уравнение включает внутреннюю индуктивность модельного рельса, но эта индуктивность была бы небольшой

, потому что материал рельса был немагнитным, а частота измерения была достаточно высокой, чтобы проникновение тока

было небольшим по сравнению с размерами рельса. (скин-эффект).Таким образом, предполагается, что приведенное выше уравнение

дает близкое приближение к внешней индуктивности LE рельсов.

A3.2. Магнитная проницаемость гусеницы

Для полноразмерной гусеницы индуктивность будет больше, чем у модели, поскольку магнитная проницаемость гусеницы

немного больше единицы. Эта повышенная проницаемость обусловлена ​​наличием металла между рельсами, а не

Как ведет себя индуктор в цепи переменного тока? — Mvorganizing.org

Как катушка индуктивности ведет себя в цепи переменного тока?

Следовательно, когда на катушку индуктивности подается синусоидальное напряжение, оно опережает ток на одну четверть цикла или на фазовый угол 90º.Ток отстает от напряжения, поскольку индукторы противодействуют изменению тока. Изменение тока вызывает ЭДС. Это считается эффективным сопротивлением катушки индуктивности переменному току.

Как индуктор работает в цепи?

Катушка индуктивности — это пассивный электронный компонент, способный накапливать электрическую энергию в виде магнитной энергии. По сути, он использует проводник, намотанный на катушку, и когда электричество течет в катушку слева направо, это создает магнитное поле в направлении по часовой стрелке.

Для чего нужен индуктор?

Катушки индуктивности используются в качестве накопителя энергии во многих импульсных источниках питания для выработки постоянного тока. Катушка индуктивности подает энергию в схему для поддержания протекания тока в периоды выключения и позволяет создавать топографии, в которых выходное напряжение выше входного.

Что будет, если конденсатор заменить катушкой индуктивности?

Внешний индуктор, заменяющий конденсатор, не изменяет фазовый угол тока в катушке запуска / работы — это будет та же фаза (или около того), что и ток в основной катушке, и вращение поля не будет.

Конденсаторы удерживают напряжение?

Максимальное напряжение — Каждый конденсатор имеет максимальное напряжение, с которым он может работать. В противном случае он взорвется! Вы найдете максимальное напряжение от 1,5 до 100 В. Эквивалентное последовательное сопротивление (ESR) — Как и любой другой физический материал, клеммы конденсатора имеют очень маленькое сопротивление.

Что происходит с конденсатором на высокой частоте?

По мере увеличения частоты конденсатор пропускает через пластины больше заряда за заданное время, в результате чего через конденсатор протекает больший ток, как если бы внутренний импеданс конденсатора уменьшился.

Как работает конденсатор на низких частотах?

На низких частотах достаточно времени для накопления значительного количества заряда на конденсаторе. Это означает, что на конденсаторе имеется большое «обратное напряжение», поэтому сумма напряжения конденсатора и напряжения источника переменного тока почти равна нулю. Это означает, что ток будет почти нулевым.

Какой конденсатор используется для высокой частоты?

Конденсаторы керамические

Как катушка индуктивности ведет себя на низких частотах?

При низкой частоте реактивное сопротивление падает; при постоянном токе индуктор ведет себя как короткое замыкание.С увеличением частоты реактивное сопротивление увеличивается, и при достаточно высокой частоте реактивное сопротивление приближается к реактивному сопротивлению разомкнутой цепи.

Влияет ли индуктор на частоту?

Индуктивное реактивное сопротивление катушки индуктивности увеличивается с увеличением частоты на ней, поэтому индуктивное реактивное сопротивление пропорционально частоте (XL α ƒ), поскольку обратная ЭДС, генерируемая в катушке индуктивности, равна ее индуктивности, умноженной на скорость изменения тока в катушке индуктивности. .

Есть ли у индуктора сопротивление?

В реальной жизни индуктор состоит из катушки с проволокой (с многослойным железным сердечником или без него).Таким образом, настоящая катушка индуктивности имеет как сопротивление, так и индуктивность. Если вы удвоите индуктивность, увеличив длину провода на катушке, сопротивление увеличится (примерно в 1,4 раза).

Как частота влияет на цепь переменного тока?

В индуктивности, чем ниже частота, тем меньше ее полное сопротивление. Таким образом, при постоянном напряжении на нем ток будет расти, если частота станет ниже. Во-первых, нам нужно понять компоненты управления скоростью двигателя переменного тока. Для управления скоростью двигателя переменного тока требуется соотношение входного напряжения / частоты для управления скоростью двигателя.

Как изменяется индуктивное сопротивление при увеличении частоты?

Как повлияет на индуктивное сопротивление XL, если частота источника переменного тока увеличится?

Индуктивное реактивное сопротивление XL = ωL = 2πvL будет увеличиваться с увеличением частоты v, а емкостное реактивное сопротивление XC = ωC1 = 2πvC1 будет уменьшаться с увеличением частоты v.

Конденсаторы блокируют переменный или постоянный ток?

Конденсатор действует как эластичная мембрана, он допускает колебания, но блокирует прохождение постоянного тока.

Допускает ли индуктор переменного тока?

Катушка индуктивности накапливает электрическую энергию в виде магнитной энергии. Катушка индуктивности не позволяет переменному току проходить через нее, но позволяет постоянному току проходить через нее.

Что блокирует переменный ток и пропускает постоянный ток?

Конденсаторы блокируют постоянный ток и создают сопротивление переменному току, пропорциональное величине конденсатора и обратно пропорциональное частоте. Катушки индуктивности пропускают постоянный ток и создают сопротивление переменному току, пропорциональное величине индуктивности и частоте.

Почему конденсатор работает только от сети переменного тока?

Реактивное сопротивление емкости обратно пропорционально частоте. Для источника постоянного тока, когда частота равна нулю, реактивное сопротивление емкости равно бесконечности. поэтому емкость ведет себя как разомкнутая цепь для питания постоянного тока. Таким образом, емкость будет работать только для питания переменного тока.

Почему индуктор не используется в постоянном токе?

Катушка индуктивности представляет собой пассивную цепь. При подаче постоянного тока на катушку индуктивности он действует как короткое замыкание. Когда в индукторе используется постоянный ток, магнитный поток не изменяется, поскольку постоянный ток не имеет нулевой частоты.…

Почему переменный ток опаснее постоянного тока?

Считается, что переменный ток

более опасен, чем постоянный, потому что среднеквадратичное значение переменного тока намного больше, чем его первоначальное значение. Наше сердце приводится в движение электрическими импульсами; высокая электрическая частота переменного тока может повлиять на частоту сердечных сокращений и привести к сердечному приступу.

Цепи переменного тока серии

— Engineer-Educators.com

Если цепь переменного тока состоит только из сопротивления, значение импеданса такое же, как и сопротивление, и закон Ома для цепи переменного тока, I = E / Z, точно соответствует то же, что и для цепи постоянного тока.На рисунке 126 проиллюстрирована последовательная цепь, содержащая лампу с сопротивлением 11 Ом, подключенную к источнику. Чтобы определить, сколько тока будет протекать, если приложено 110 В постоянного тока и сколько тока будет протекать, если приложено 110 В переменного тока, решаются следующие примеры:

Когда цепи переменного тока содержат сопротивление и либо индуктивность, либо емкость, полное сопротивление Z, не то же самое, что сопротивление R. Импеданс цепи — это полное сопротивление цепи протеканию тока. В цепи переменного тока это противостояние состоит из сопротивления и реактивного сопротивления, индуктивного или емкостного, либо их элементов.

Сопротивление и реактивное сопротивление нельзя сложить напрямую, но их можно рассматривать как две силы, действующие под прямым углом друг к другу. Таким образом, соотношение между сопротивлением, реактивным сопротивлением и импедансом можно проиллюстрировать прямоугольным треугольником. [Рисунок 127]

Рисунок 127. Треугольник импеданса.

Поскольку эти величины могут быть связаны со сторонами прямоугольного треугольника, формулу для определения импеданса или полного противодействия току в цепи переменного тока можно найти, используя закон прямоугольных треугольников.Эта теорема, называемая теоремой Пифагора, применима к любому прямоугольному треугольнику. В нем говорится, что квадрат гипотенузы равен сумме квадратов двух других сторон. Таким образом, значение любой стороны прямоугольного треугольника можно найти, если известны две другие стороны. Если цепь переменного тока содержит сопротивление и индуктивность, как показано на рисунке 128, соотношение между сторонами может быть указано как:

Рисунок 128. Схема, содержащая сопротивление и индуктивность.

Корень квадратный из обеих частей уравнения дает

Эту формулу можно использовать для определения импеданса, если известны значения индуктивного реактивного сопротивления и сопротивления. Его можно изменить для определения импеданса в цепях, содержащих емкостное реактивное сопротивление и сопротивление
, путем замены XC в формуле. В цепях, содержащих сопротивление как с индуктивным, так и с емкостным реактивным сопротивлением, реактивные сопротивления можно комбинировать, но поскольку их эффекты в цепи прямо противоположны, они объединяются вычитанием:

На рисунке 128 последовательная цепь, состоящая из сопротивления и индуктивности, соединенных между собой. последовательно подключается к источнику 110 вольт при 60 циклах в секунду.Резистивным элементом является лампа с сопротивлением 6 Ом, а индуктивным элементом — катушка с индуктивностью 0,021 Генри. Какое значение импеданса и тока через лампу и катушку?

Решение:

Сначала вычисляется индуктивное реактивное сопротивление катушки:

Затем вычисляется полное сопротивление:

Затем протекает ток,

Падение напряжения на сопротивлении (E R ) равно

Падение напряжения на индуктивности (EX L ) равно

Сумма двух напряжений больше приложенного напряжения.Это происходит из-за того, что два напряжения не совпадают по фазе и, как таковые, представляют максимальное напряжение. Если напряжение в цепи измеряется вольтметром, оно будет примерно 110 вольт, подаваемое напряжение. Это может быть доказано уравнением

На рисунке 129 проиллюстрирована последовательная схема, в которой конденсатор емкостью 200 мкФ соединен последовательно с лампой на 10 Ом. Какое значение имеет импеданс, ток и падение напряжения на лампе?

Рисунок 129. Цепь, содержащая сопротивление и емкость.
Решение:

Сначала емкость изменяется с микрофарад на фарады. Так как 1 миллион микрофарад равен 1 фараду, тогда

Чтобы найти импеданс,

Чтобы найти ток,

Падение напряжения на лампе (E R ) составляет

Падение напряжения на конденсаторе (EX C ) равно

Сумма этих двух напряжений не равна приложенному напряжению, поскольку ток опережает напряжение.Чтобы найти приложенное напряжение,

Когда цепь содержит сопротивление, индуктивность и емкость, для определения полного сопротивления используется уравнение

.

Пример: Каков импеданс последовательной цепи, состоящей из конденсатора с реактивным сопротивлением 7 Ом, катушки индуктивности с реактивным сопротивлением 10 Ом и резистора с сопротивлением 4 Ом? [Рисунок 130]

Рисунок 130. Цепь, содержащая сопротивление, индуктивность и емкость.
Разное

Добавить комментарий

Ваш адрес email не будет опубликован. Обязательные поля помечены *